You are on page 1of 88

A Course in English Syntax

Syllabi for the Lectures


Examples and Excercises

Ludmila Veselovsk

TABLE OF CONTENTS
INTRODUCTION ............................................................................................................................................ 4
Working with this text................................................................................................................................ 4
The Topics and Background Philosophy.................................................................................................. 5
1

CLAUSE .................................................................................................................................................. 1
1.1
MODELS OF PREDICATION (CLAUSE STRUCTURE) ............................................................................ 1
1.1.1
Relation between Subject and Predicate..................................................................................... 1
1.2
OTHER SENTENCE MEMBERS............................................................................................................. 3

SUBCATEGORISATION ...................................................................................................................... 4
2.1.1
Lexical items: Subcategorization ................................................................................................ 4
2.1.2
Hierarchies of semantic roles and grammatical functions......................................................... 5
2.1.3
English Verbs with respect to their syntactic subcategorization................................................. 6
2.2
COMPLEMENTS AND ADJUNCTS ...................................................................................................... 7
2.2.1
Defining Complements and Adjuncts .......................................................................................... 7
2.2.2
Adjuncts: Verbal and Sentential Adverbs ('Scope')..................................................................... 8
2.3
EXERCISES ...................................................................................................................................... 10

OBJECTS .............................................................................................................................................. 14
3.1
3.2
3.3
3.4

SEMANTIC ROLES OF OBJECTS ....................................................................................................... 14


MORPHOLOGICAL PROPERTIES OF OBJECTS ................................................................................... 14
SYNTACTIC DEFINITION OF OBJECTS (COMPLEMENTS) .................................................................. 15
EXERCISES ...................................................................................................................................... 16

PASSIVIZATION ................................................................................................................................. 18
4.1
4.1.1
4.1.2
4.2
4.2.1
4.2.2
4.3
4.4

FUNCTION (PRAGMATICS/USAGE) OF PASSIVIZATION ................................................................... 18


Demoting the Subject/Agent of the active Verb = Deagentization............................................ 18
Focusing the Agent which becomes 'by-' Adjunct .................................................................... 19
RESTRICTIONS ON PASSIVIZATION .................................................................................................. 20
Categorial incompatibility ........................................................................................................ 20
Semantic Restrictions................................................................................................................ 20
A REMARK ABOUT SEMANTIC NOTIONS OF ACTION VERSUS STATE ............................................... 21
EXERCISES ...................................................................................................................................... 22

COMPARING CZECH AND ENGLISH PASSIVES ....................................................................... 23


5.1
THE DOUBLE OBJECT STRUCTURES................................................................................................ 23
5.2
PASSIVIZATION WITH VERBAL COMPLEXES (RESTRUCTURALIZATION).......................................... 25
5.2.1
Phrasal Verbs (Preposition Stranding).................................................................................... 25
5.2.2
Verbo-nominal phrases : complex Predicates .......................................................................... 26
5.2.2.1

5.3
5.4
6

SUBJECT............................................................................................................................................... 31
6.1
6.2
6.2.1
6.2.2
6.2.3
6.3
6.4

V+N Compounds ...........................................................................................................................27

MEDIOPASSIVE ............................................................................................................................... 28
EXERCISES ...................................................................................................................................... 28

SEMANTIC ROLES OF SUBJECTS ...................................................................................................... 31


MORPHOLOGICAL PROPERTIES OF SUBJECTS .................................................................................. 32
Subject Case in English (Nominative)....................................................................................... 32
Subject Predicate Agreement ................................................................................................. 33
Pro-Drop Languages ................................................................................................................ 33
PRAGMATIC ROLES OF SUBJECTS (RELATED TO LINEARITY)........................................................... 34
EXERCISES ...................................................................................................................................... 34

SYNTACTIC PROPERTIES OF ENGLISH SUBJECTS ................................................................ 36


7.1
DISSOCIATING THE FORM AND THE MEANING ................................................................................. 37
7.1.1
Dissociated Subject (Expletives there and it)............................................................................ 37

7.1.2
Locatives / Directional structures............................................................................................. 38
7.1.3
Raising to Subject ..................................................................................................................... 39
7.2
EXERCISES ...................................................................................................................................... 40
8

ATTRIBUTE ......................................................................................................................................... 42
8.1
8.2
8.3
8.4

PRENOMINAL ATTRIBUTES ............................................................................................................. 42


POSTNOMINAL ATTRIBUTES ........................................................................................................... 43
THE RELATION BETWEEN THE NOUN AND ITS ATTRIBUTES............................................................ 44
EXERCISES ...................................................................................................................................... 44

NEGATION ........................................................................................................................................... 47
9.1
9.1.1
9.1.2
9.2
9.3
9.4
9.5
9.5.1
9.5.2
9.6
9.7

10

THE SCOPE OF NEGATION............................................................................................................. 55


10.1
10.2
10.3
10.4
10.5

11

KINDS OF NEGATION ...................................................................................................................... 47


Semantic Negation .................................................................................................................... 47
Partial Negation (Lexical, phrasal) .......................................................................................... 47
SENTENCE NEGATION ..................................................................................................................... 48
THE FORM AND NUMBER OF NEGATIVE OPERATORS........................................................................ 48
THE CHOICE OF THE UNIQUE NEG IN ENGLISH .............................................................................. 49
POSITION OF NEGATION IN ENGLISH (REVISION) ............................................................................ 49
Declarative sentences ............................................................................................................... 49
Negative Questions ................................................................................................................... 50
NOT VS. NEVER ........................................................................................................................... 51
EXERCISES ...................................................................................................................................... 52

SHORTENING THE SCOPE OF NEG ................................................................................................... 55


ENLARGING THE SCOPE OF NEG (NEG-TRANSPORTATION/TRANSFER) ......................................... 55
NEGATIVE ADVERBS / PARTIAL NEGATION .................................................................................... 56
MORE POLARITY ITEMS .................................................................................................................. 56
EXERCISES ...................................................................................................................................... 57

ENGLISH SENTENCE PATTERNS (STEP BY-STEP)................................................................ 60


11.1
ANALYTIC CHARACTERISTICS OF THE ENGLISH PREDICATE ........................................................... 60
11.1.1 Negation.................................................................................................................................... 60
11.1.2 Polar Questions: ....................................................................................................................... 60
11.1.3 Verb 'be' and single lexical Verbs ............................................................................................. 61
11.1.4 Other use of the structure S-M/A-V-(O).................................................................................... 62
11.1.5 Negative questions .................................................................................................................... 63
11.2
WH QUESTIONS .............................................................................................................................. 64
11.2.1 Indirect WH questions and Echo Questions.............................................................................. 65
11.3
IMPERATIVES (COVERT SUBJECT)................................................................................................... 65

12

SUBORDINATE CLAUSES ................................................................................................................ 66


12.1
CLASSIFICATION OF SUBORDINATE CLAUSES ................................................................................. 66
12.2
NOMINAL CONTENT CLAUSES ........................................................................................................ 67
12.2.1 Resumptive element ('oprn vraz')......................................................................................... 67
12.2.2 Tense/Mood (and other characteristics) of the Subordinate Clause......................................... 67
12.3
ADVERBIAL CONTENT CLAUSES..................................................................................................... 68
12.3.1 Kinds of Adverbial content clauses ........................................................................................... 68
12.3.2 Conditional Clauses.................................................................................................................. 68
12.3.2.1
12.3.2.2
12.3.2.3
12.3.2.4

12.4
13

Future Conditional Clauses.............................................................................................................69


Present Conditional Clauses............................................................................................................69
Past Conditional Clauses.................................................................................................................69
Mixed Patterns ................................................................................................................................69

EXERCISES ............................................................................CHYBA! ZLOKA NEN DEFINOVNA.

RELATIVE CLAUSES......................................................................................................................... 69
13.1.1
13.1.2
13.1.3

Kinds of Relative Clause ........................................................................................................... 70


Relative Clause Formation ....................................................................................................... 70
Relative Pronoun Deletion........................................................................................................ 71

13.1.3.1

Preposition stranding in Relative clauses........................................................................................71

13.1.4 Content Nominal/Attributive vs. Relative Clauses .................................................................... 72


13.2
EXERCISES ...................................................................................................................................... 73
14

COMPLEX SENTENCE ANALYSIS .............................................................................................. 77


14.1
14.2
14.3
14.4

PARTS OF SPEECH (WORD CATEGORIES).......................................................................................... 77


SEMANTIC ROLES: PARTICIPANS OF THE VERBAL EVENT/ARGUMENTS OF THE VERB.................... 77
SENTENCE FUNCTIONS (SENTENCE MEMEBERS, SYNTAGMA).......................................................... 78
SENTENCE DYNAMISM / FUNCTIONAL SENTENCE PERSPECTIVE ..................................................... 79

TOPICS TO DISCUSS ................................................................................................................................... 81


RELATED LITERATURE............................................................................................................................ 82

INTRODUCTION
Working with this text
This text has been written to assist students of English in their work in the course of
English Syntax in the programme of English philology. It assumes a solid knowledge of
English grammar, especially of morpho-syntactic properties of English parts of speech and
their grammatical categories. The latter topics are listed and covered in the Course in
English Morphology and Morphosyntax (Veselovsk 2008). Some data and phenomena
discussed in that volume are repeated here and analyzed from a distinct perspective.
This text, however, is in no way intended to replace any textbook specified in a
course description, nor does the amount of material cover all of what students need to read
for their exams. Instead, it provides syllabi for the lectures with many schemes and
examples commented and discussed in the course. Without a commentary some of them
may be difficult to understand and the students are strongly encouraged to make their own
notes and remarks during the classes. Enough space is given between the paragraphs and on
the margins so that such additions were possible. Some students may still have problems
with English terminology and structuring of their study - this text should also provide them
with the main terms used, and the sections basically follow a pattern that can be used in
preparing for English grammar exams, though not all topics are covered to the same extend
and some require more individual reading.
Apart from syllabi, the following text also contains a number of exercises. The
function of the exercises is twofold. First, they introduce some new Aspects or problems of
the proposed analyses not mentioned in detail during the lectures. Second, they allow
students to test their understanding of the topics under discussion. For some suggested
problems, naturally, there is no generally agreed solution and the exercise provides more
data for discussion of alternatives than simple minded test of a knowledge.

The Topics and Background Philosophy


The text in this volume is divided into two parts can be covered in some 10-13 twohour classes (in the existing system in a semester). It constitutes a general introduction to
the study of simple clauses, deals with the sentence members and their semantic,
morphological and syntactic properties in English. At the end of this section the form of
English clausal negation is introduced and demonstrated together with the English main
sentence patterns (both simple and complex) classified according to their function and
form.
Each Chaptert contains introductory Revision section testing the assumed preliminary
knowledge, and a final Revision section which summarizes the basic topics covered in the
course and required for the final exam.
The text concentrates on topics which the author finds most important, most
interesting and sometimes neglected in other study materials. To complement these
individual choices, at the beginning of most sections there are some bibliographical
references to the literature which are recommended as study material for the course. The
students are expected to go through at least some of these materials mentioned.
Because the assumed readers are Czechs and many of them intend to translate or
interpret in their future careers, English grammar is usually compared with its Czech formal
and/or pragmatic equivalents. Some other languages are also occasionally mentioned, to
provide a more universal background for the topic under discussion.
The author of the following text believes in linguistics, above all in grammar, as an
autonomous science. Therefore the analyses here assume that human language is a system
which can be studied applying scientific methods with the result of acquiring some
descriptively adequate and as explanatory as possible generalized hypotheses. Empirical
data and argumentation are thus strongly preferred to the memorizing of any listed
classifications, and no a priori analysis or theory is taken for granted or as definitive.
Nonetheless, the presentation and hypotheses here, such as in the choices for categories, are
based on traditional functional and structuralist grammar (which the students used during
their pre-university grammar education) only slightly influenced by current theoretical
proposals.
Recent functional and generative approaches typically present themselves as returning to the empirical concerns of traditional grammar and at the moment provide a wide
range of plausible frameworks. The grammatical analyses introduced in this couse assume
the need of empirical and scientific understanding of human language and although it
concentrates on formal grammar, it assumes interactions with other disciplines such as
theory of communication, literary study, psychology, sociology, anthropology, etc. The
author hopes that discussing and trying to understand basic grammar in a more universal
and open-minded way turns out to be useful for all students of English language, who can
then go on in their studies in whichever field or framework suits their fancy.
Ludmila Veselovsk

CLAUSE

H&P (2002) pp. 235-243 ; QGLS (2004) pp. 717-770; Dukov (1994) pp.401-422;
H&P (2005) pp.11-28, pp. 63-66; G&Q (1991) pp. 204-230;
1.1

Models of Predication (Clause Structure)

Clause:

Clause is a pronounced or written idea...


It says something about something...
It consists of Subject and Predicate...

(a)
(b)

semantic notion of the (kind of) predication relation


formal means to express the two main parts of the sentence and the relation(s)
between them

(1)

Revising terminology

(I) SIMPLE
(II) NON-SIMPLE (A) COMPLEX = one main + 1-x subordinate clause(s)
(B) COMPOUND
(C) COMPLEX COMPOUND
= more than one main + some subordinated clause(s)
(III) SEMI-CLAUSE = infinite Predicate
(a)
(b)
(c)
(d)
(e)
(2)

Yesterday Emma saw William in the living room.


When Emma arrived she saw William in the living room.
Emma arrived yesterday and saw William in the living room.
When Emma arrived yesterday she saw William in the living room and Bill in the
kitchen which was full of steam and she heard the sound of the running dishwasher.
I saw [ Joe leaving the house at 5 o'clock. ]
(a)
(b)
(c)
(d)

This sentence is simple/complex.


This clause is simple / ??? complex.
This sentence consists of three clauses.
* This clause consists of three sentences/clauses.

1.1.1 Relation between Subject and Predicate


The relation between the two main sentence members (Subject and Predicate) is defined
depending on the framework, i.e. the model of grammar used (esp. the notion of a
hierarchy: what is the main element). In all models, however, the structureis defined in
terms of constituents bigger than 'word' (Subject/Object etc. are phrases)
(3)

Subject and Predicate as equal members. (A traditional view.)


John
SUBJECT

reads a book.
PREDICATE
Verb +complementation

PREDICATION =
The relation between the
SUBJECT and the
PREDICATE

(4)

Verb as the main memeber of the structure. Typical for all kinds of valency models.
XP
(Subject)
left valency

John
My little sister Barbara

(5)

YP
(Object?)
right valency

reads
couldn't have given

a book
the bunch of flowers to her boy-friend.

Immediate Constituent Analysis


Structuralist model. The scheme/tree is able to capture

(a)
(b)

categorial labels
syntactic relations

S
NP
[Subject]
John

Mod/Aux (NEG)

VP
[Predicate]

will
V
read

XP
[V-complementation]
a book

Immediate constituent analysis captures also the fact that the clause consists of more
complex units (phrases) which can be further divided into smaller units. The tree can be
therefore enlarged to the level of individual words (heads).
(6)

(a)
(b)

the new book of some folk stories


[NP the new book [PP of [NP some folk stories]]]

NP
Det
the

PP
NP

AP
new

NP
N
book

(7)

P
of

NP
Q
some

PP
of some folk stories

NP
N(P?)
folk

N
stories

Linear notation (for practical purposes)


English sentence:

SUBJ
John

Aux/Mod (NEG) V
does
(not) read
will
can
/-s

+ YP
the book

1.2

Other sentence members

The clausal structure is a hierarchy build of binary relations. These relations = functions =
syntagmas have usually two members (one higher/superordinate) and the other dependent
(subordinate). The sentence members are therefore relational terms.
(8)

(a)
(b)
(c)

similarly

a flower
a sister
a widow
(d)
(e)

(independent expression)
(assumes/requires the existence of a brother or sister)
(assumes merriage and the death of the partner.

Noun
Subject

(the property of the expression itself)


(the expression when related to predicate)

(9)

A constituent in a clause can thus be classified both independently and in a relation:

(a)
(b)

part of speech (phrase)


sentence memeber

(10)

(independent classification of the element itself)


(in relation to its syntagmatic partner)

Our Mary arrived late.

Our Mary is

(a) NP
(b) a Subject of arrived

(11) Syntagmatic relations:


(a)
(b)
(c)
(d)
(e)
(f)

Subject
Predicate
Object
Adverbial
Attribute
Complement

(related to Predicate)
(related to Subject)
(related to Verb or Preposition)
(related to Verb)
(related to Noun)
(related to both Verb and Noun)

John arrived.
Bill slept.
(to) see/ about Mary
(to) arrive late/in the afternoon
big Ben, brother of mine
John arrived tired

The structure can be analysed at distinct levels of complexity


(12) John lives in his father's house.
(a)
(b)
(c)

in his father's house:


his father's house:
father's:

PP, adverbial of place related to the verb lives


NP, object of the preposition in
NP, attribute to house

(d)

John [VP lives [PP in [NP his father's [N house]]]].

(13) The trenary relation of Subject/Object Complement


(a)
(b)
(c)

Mary painted the door.


The green door...
Mary painted the door green.

- door is an object of painted


- green is an attribute of door
- green is a complement of painted and door

(d)
(e)

my tired brother
My brother came back tired.

- tired is an attribute of brother


- tired is a complement of brother and came back

SUBCATEGORISATION

H&P (2002) pp. 213-319, pp. 663-784; QGLS (2004) pp. 740-754; pp. 1147-1234;
Dukov (1994) pp. 349-367; H&P (2005) pp.63-81; G&Q (1991) pp. 336-362;
In the scheme in (5) on page 2 above, the Subject is labeled as NP (which it often is) and
the right hand verbal complementation is labeled as XP. What is it the XP (i.e. what follows
the V, creating VP/Predicate? Notice that some of these XPs are obligatory.
(1)

(a)
(b)
(c)
(d)
(e)
(f)
(g)
(h)
(i)

Jane sent a parcel to her mother.


Jane sent a parcel.
*Jane sent.
Wilhelm ran to the park early in the morning.
Wilhelm ran to the park.
??Wilhelm runs.
Bill can arrive soon.
Bill can arrive.
?? Bill can.

2.1.1 Lexical items: Subcategorization


(2)

REVISION: Valency of the Verb: (Participants/Arguments of the Verb)


verbal event
action

1st participant
(Agent)
(3)

(a)
(b)

Peter/He
Petr/On

complementary conditions
(Manner/Place/Time)
2nd participant
(Patient)

sent
poslal

3rd participant
(Recipient/Beneficiary)

a parcel/it to Henry/to him in the afternoon


balk
Janovi
v poledne

Verbal subcategorization states the (form of) complementation required by the Verbs.
(4)

Semantic subcategorisation of the Verb hit :

hit: <Agent, Patient>

(5)

Formal/Syntactic subcategorisation
of the Verb hit :

hit: V, [-- NP]

for its full interpretation


the activity hit requires two
Arguments (with the semantic
roles of Agent and Patient )

the lexical item hit is a Verb


and it must be complemented by
a nominal element (NP, Object),
which canonically follows it.

2.1.2 Hierarchies of semantic roles and grammatical functions


The relation between the meaning (semantic role) and the form of the constituent is subject
to both universal and language specific rules.
(6)

Petr/On (in (3) above)


(i) Semantic role
(ii) Sentence function
(iii) Formal marking of the sentence member

Agent
Subject (usually)
Nominative/pre-Verbal, etc. .

The semantic roles are the properties of a specific lexical Verb ("follow from its
meaning"), they form a hierarchy. Also the sentence functions (sentence members) form a
hierarchy. The two hierarchies (semantic and grammatical) are related/matched, which
results in the semantic interpretation of the grammatical functions.
(7)

THE FORM AND THE MEANING


Canonic Formal Realization of the Semantic Roles (with active Verb)
Semantic hierarchy
(Arguments: Semantic Roles)

(a)
(b)
(a)
(b)

Agent
Patient/Goal
Recipient/Beneficiary
Direction/Manner/etc.

A 1 ....................
A 2 ....................
A 3 ....................
A 4 ....................

Formal hierarchy
(Sentence Functions/Members)
SF 1
SF 2
SF 3
SF 4

= / Subject
= / Direct Object
= / Indirect Object / PP
= / PP / Adv

The two hierarchies cannot be unified into one, because the relation between them depends
on many factors and can be language/speaker specific (especially with Verbs of nontypical
semantic roles). The formal hierarchy is language specific and the semantic hierarchy
depends on a specific lexical item and can be to some extend modified by the speakers
analysis of the event.
(8)

The influence of the verbal form (active vs. passive Verb)

(a)
(b)
(c)

She wrote many letters to Bill.


Many letters were written to Bill (by her/Emma)
Bill was written many letters (by her/Emma)

(9)

Language specific modifications of the (untypical) semantic hierarchy


'like'
.... who is (the active) Agent? who is (the affected) Patient?
(a)
(b)

I like John.
Jan se mi lb.

...English choses I for the top semantic role.


...Czech choses Jan for the top semantic role.

The knowledge of a language includes


(i)
(ii)

the knowledge of the formal hierarchy of the sentence functions/members and


the properties of specific lexical and grammatical unites/items including their
semantic hierarchies.

(10) Information about each lexical item contains


a)
b)
c)

semantic definition (what does the word mean, e.g. borrow = get temporarily...)
phonetic information (how does it sound, e.g. borrow = ['boru])
syntactic information
i.) which category is it (e.g. borrow is a Verb)
ii) subcategorization (borrow requires Complement)

2.1.3 English Verbs with respect to their syntactic subcategorization


The Verbs can be classified w.r.t. to the number and characteristics of their Complements,
i.e. w.r.t. the constituents they require (i.e. which are obligatory!) to appear with them to
form a grammatically complete structure (not be mistaken for communicative
completeness/appropriateness).
(11) (a)

*Peter wrote.

(b)
(c)
(d)

Peter wrote a letter.


Peter wrote a letter to Mary.
Peter wrote a letter in the kitchen late in the afternoon.

(12) Subcategorization of English Verbs (traditional, functional-structuralist approach)


Subcategorization*

Example

Traditional term

The boy can come.


Joe was a student.
Joe is being silly.
Joe must be at home.

Modal
Copula
linking Verb

Aux/Mod, [-- VP]


Aux/V, [-- NP]
Aux/V, [-- AP]
Aux/V, [-- PP]

1.
2.
3.
4.

V, [--]
V, [--NP]
V, [-- PP]
V, [-- AP]
V, [-- NP NP]
V, [-- NP PP]
V, [-- NP PP/Adv]
V, [-- NP NP]
V, [--NP AP]

5. The tramp laughed.


6. Mary read a diary.
7. The tramp leaned towards the girl.
8. He seemed tired.
9. He told a girl an interesting story.
10. He wrote a letter to Joe.
11. Joe put a book on the shelf.
12. He called her a clever girl.
13. The music drives me mad.

intransitive V
monotransitive V
V of movement
ditransitive V
<Patient, Beneficiary>
<Patient, Location>
complex trans. V

Notice that - all the verbal Complements are defined as phrases,


- some Vs have VP Complements, as in (1) in the table above,
- many Complements can be replaced by a semiclause (infinitive/-ing form) or a clause.
(13) (a)
(b)

I am writing the paper. / I have written the paper.


I can write the paper. / I must have written the paper.

(14) (a)
(b)
(c)
(d)
(e)

I started/finished/wanted to write the paper.


I love/hate to write the paper / writing the paper.
I saw/heard Benjamin write the paper / writing the paper.
He said that Mary would come in time.
He asked whether Mary would come in time.

2.2

Complements and Adjuncts

Compare the distinction between the way a language signals the grammatical function:
- in Czech it is mainly the Case (i.e. morphology),
- in English it is the word order (constituent order w.r.t. the Predicate).
(1)

(a)
(b)

JanNOM psal knihuACC.


KnihaNOM je psna JanemINSTR.

(a')
(b')

SheSUBJ saw himV-OBJ.


HeSUBJ was seen by herP-OBJ.

In English many Czech "Objects" have Prepositions. How shall we call the underlined
constituents? Do "Objects" have Prepositions?
(2)

(a)
(b)
(c)
(d)
(e)
(f)
(g)

Hrabala zahradu hrbmi.


Hrabala zahradu s hrbmi.
Pinesl mamince kytiku.
Pinesl pro maminku kytiku.
Dvala se na obrzek.
Zpvala pro publikum.
Dal to Pavlovi.

(a'/b') She raked the garden with a rake.


(c'/d') He brough a flower for his mother.
(e')
(e')
(g')

She was looking at the picture.


She was singing for the audience.
He gave it to Paul.

The traditional (Latin based) concept of Object depends to large extend on morphology.
The term can become vague when applied on analytic English. In English therefore a
distinct terminology is sometimes used, namely Complement (approx.Object) and Adjunct
(approx. Adverbial).
2.2.1 Defining Complements and Adjuncts
The distinctions between Complements and Adjuncts can be illustrated in both semantic
and formal level. The terms, however, are primarily syntactic.
A.

Lexical & Semantic 'closeness':


Complements are semantically related to the verbal action (they are participants
necessary for the event to happen) while Adjuncts are more independent.

(3)

(a)
(b)
(c)
(d)

B.
-

Formal obligatoriness
OBLIGATORY complementation of a Verb is called Complement.
OPTIONAL complementation of a Verb is called Adjunct.

(4)

(a)
(b)
(c)
(d)

C.

The Number of Complements and Adjuncts:

write a letter /* a tree


kill a man /* a stone
wrote/killed sb/st in January
to rely (*)/ ON nuclear weapons

Mary must send a letter to Henry on Monday.


Mary must send a letter on Monday.
*Mary must send to Henry on Monday.
*Mary must send.

Complements are unique, while Adjuncts can occur in larger numbers.


*read [a book] [a journal]
read [a book] [at home] [yesterday] [for two hours] ....

(5)

(a)
(b)

D.

Distribution: neutral word-order:


Complements immediately follow the Verb. Adjucts are more peripheral.

(6)

(a)
(b)
(c)
(d)
(e)

Emily will visit [Rome] [(on) Tuesday]


* Emily will visit [on Tuesday] [Rome]
Hilary wrote [a letter] [in January] [at home]
Hilary wrote [a letter] [at home] [in January].
* Hilary wrote [at home] [a letter] [in January]

(f)
(g)

student [of Linguistics] [with curly hair]


* student [with curly hair][of Linguistics]

2.2.2 Adjuncts: Verbal and Sentential Adverbs ('Scope')


Quirk/Greenbaum/Leech/Svartvik (2004) pp. 475-654, 729-738;
Appart from its Complements (cca. Objects), the Predicate is often modified with respect to
other parameters: Manner, Time, space etc.
(7)

(a)
(b)
(c)
(d)

He runs quickly. He does not speak naturally.


Naturally, he can speak English
He will not perhaps speak.
Certainly, he speaks English.

Adverbials modify some other element. They "take scope" over it.
The SCOPE of the Adverb

(a) the verbal action,


(b) YES/NO polarity or the whole proposition
(c) the whole proposition
(d) some sentence member

We can define the Complements/Adjuncts/Disjuncts also w.r.t. to the scope they take. The
terminology is related to the size/level/projection of the verbal phrase: elements take scope
over the constituent which they adjoin.
(8)
(a)
(b)
(c)
(d)
(e)
(f)
(g)

Of course, he will never speak English well in the classroom.


speak + English
[speak English] well
[speak English well] in the clasroom
never [speak English well in the clasroom]
will [never speak English well in the clasroom]
he [will never speak English well in the clasroom]
of course [he will never speak English well in the clasroom]

minimal VP
VP
VP
Predicate
sentence
sentence
sentence

(9)

The Scope of verbal/sentential complementation


(Structural view of the gradual, step-by-step verbal/sentential projection )
Of course, he will not/never speak English well in the classroom.

Sentence

DISJUNCT (s)
Of course,

sentence (Modality / Polarity)

VP
Subj. NP
He

Mod/Aux+NEG
will (not/never)

VP
VP
minimal VP
V
speak

(10) a.

b.

ADJUNCTS are adjoined inside VP (they are external w.r.t. V+Complement).


The students [VP read books every day].
I want to [VP see Jill on the picture].

DISJUNCTS take scope over the whole sentence/proposition.


(i)
(ii)

d.

COMPLEMENT
English

COMPLEMENTS are closest to the Verb, they are internal the minimal VP.
(i) The students [VP read books ].
(ii) I want to [VP see Jill].
(i)
(ii)

c.

ADJUNCT (Time)
now
ADJUNCT (Place)
here
ADJUNCT(Manner)
well

Naturally, he will help you.


Of course, he is extremely polite

CONJUNCTS are not related to VP, they modify (= take scope over) some
other sentence member).
(i)
(ii)

He did it [AdvP very well ].


He is [AdvP extremely polite ].

2.3

Exercises

(11) EXERCISE ===========================================


(i) Underline the Verbs/Predicates in the following sentences. Mark their left and
right borders. (ii) How many elements does 'Predicate/Verb' have? (iii)Which part of the
complex tends to carry grammatical categories of the Verb (Tense/Aspect, Agreement) and
which is the locus of 'lexical meaning'? Define both in terms of right/left side and w.r.t. the
categorial status of the relevant element.
(a)
(b)
(c)
(d)
(e)
(f)
(g)

Mary reads novels.


Johanna would run away with the sailor.
Mr. Todd had been shaving the customers with a real enthusiasm.
Jane was allowed to look at those pictures.
My parents are not looking forward to his arrival.
Didn't he take courage, at last?
Some people take a shower twice a day.

(12) EXERCISE ===========================================


Compare the Predicates in (11) above with their equivalents. Is there any distinction?
(Use your own examples to illustrate it). Try to express the distinction in terms of distinct
typological characteristics of the two languages.
(13) EXERCISE ===========================================
In the following table fill the labes for the relevant constituents
I have read several Hemingway's novels.
the constituent

its category
(part of speech)

its function
(sentence
member

the other
memeber of the
santagma

I
several
Hemingway's
novels
several
Hemingway's
(14) EXERCISE ===========================================
Explain the grammaticality judgments for the following examples
(a)
(b)
(c)
(d)
(e)

?? He killed a stone.
?? He killed the rabbit (??which then ran away.)
He killed a rabbit hungry/in the forest.
*He killed hungry/to the forest.
?? The stone murdered a rabbit.

10

(f)
(g)
(h)
(i)
(j)

?? The book gave Mary to Bill.


?? I put the book to Bill.
* You can rely at him.
* You ought go home.
* Mary saw each other.

(15) EXERCISE ===========================================


(i) Draw the lines suggesting which XP can combine with which V and which
combinations are ungrammatical (*).
(ii) Which constituents do the Verbs require to form a grammatical structure? Fill in the
rightmost column marking which kind of phrase is the XP.
Sentence/Clause

NP

Aux/Mod

VP
V

NP

Aux/Mod

Our Mary
The camera

will
can
must
should

SEE
ARRIVE
BE

XP
XP
(a)
(b)
(c)
(d)
(e)
(f)
(g)
(h)
(i)
(j)

this nice book


rather angry
at/to school
her
as soon as possible
for all his money
three of them
running home
to help you
though I did not want it to
happen

(16) EXERCISE ===========================================


Write the subcategorisation of the Verbs in the table 2.3. Find 3 more Verbs with the
same subcategorisation).Use them in short sentences together with their complementation.
Put the Complements into brackets and mark their category.
(a)
(b)
(c)

see
arrive
be

a.

1.
2.
3.

.....................................
.....................................
.....................................

.................
.............
.................

11

b.

1.
2.
3.

.............
.............
.............

c.

1.
2.
3.

.................
.............
.................

(17) EXERCISE ===========================================


Write the subcategorization of Vs and state what the complementing elements are.
(a)
(b)

Hugo sleeps.
Hugo sleeps in his bed.

sleep:

in his bed: ........

(c)
(d)
(e)
(f)

*Hugo sent.
Hugo sent a letter.
* Hugo sent to Bill.
Hugo sent a letter - to Bill.

sent:
a letter:
(to) Bill:

....

(18) EXERCISE ===========================================


Use the following lexical Verbs in short sentences, underline their Complements and
write down their subcategorization.
(a)
(b)
(c)
(d)
(e)
(f)
(g)
(h)
(i)
(j)

play
deliver
elect
paint
give
tell
say
put
start
order

.................................................................................
.................................................................................
.................................................................................
..................................................................................
..................................................................................
..................................................................................
..................................................................................
..................................................................................
..................................................................................
..................................................................................

.................................
.................................
.................................
................................
................................
................................
................................
................................
................................
................................

(19) EXERCISE ===========================================


Distribute the words from the sentences in (18) above to the scheme below. Use the
position of negation (NEG) as a criterion for distribution.
S
NP
[Subject]

Mod/Aux (NEG)

VP
[Predicate]
V

XP

12

(20) EXERCISE ===========================================


In the table (12) on page 6 some subcategorisations look 'similar'. The relevant examples
are repeated below. Discuss the distinctions between the bold constituents.
(b)

V, [_ NP PP]

(10) He wrote a letter [PP to Benjamin].


He gave a present [PP to Emma].
(11) Benjamin put the book [PP on the shelf].
He put the present [PP to the desk].

(a)

V, [_NP,NP]

(9)

He told a girl [NP an interesting story].


Benjamin gave Emma [NP a nice present].

(12) He called her [NP a clever girl].


The comitee appointed Bill [NP a chairman].
(21) EXERCISE ===========================================
Decide whether the underlined words are Complements or Adjuncts. Mention all the
examples you have to consider to decide. Give the subcategorization of the Verb as
suggested in table in (12) on page 6.
(a)
(b)
(c)
(d)
(e)
(f)
(g)

Adam has read the article.


Adam has read the article to Mary.
Marion went to the cinema.
Peter was running to school.
Lillian saw the elephant in the ZOO.
My brother has put the papers at the top shelf.
James Bond made Goldfinger to reveal his plan.

(22) EXERCISE ===========================================


Discuss the scope of the underlined Adverbials. (What do the Adverbials modify?
What are they related to?)
(a)
(b)
(c)
(d)
(e)
(f)
(g)
(h)
(i)
(j)

Mary often comes late.


Peter was unbeliavably tired.
The frog will become a Princess, for sure.
The cake will never be finished in time.
The rabbit run away as quickly as he could.
The dream came soon true everywhere.
What are you doing overthere?
Surprisingly, little Adam has a rather American accent.
Mrs. Lovelace can speak Spanish more fluently than her son.
Soon, Mary will arrive and things become much better.

(23) EXERCISE ===========================================


(i)
(ii)

Make questions asking about the underlined Adverbs in (22) above.


Which kind of WH-element is required?
What is the second member of a relevant paradigm?

13

OBJECTS

H&P (2002) pp. 244-250; QGLS (2004) pp. 717-754; Dukov (1994) pp. 423-444;
H&P (2005) pp.63-81;
In spite of the fact that many modern English grammars use the terminology of
Complements vs. Adjuncts/Disjuncts etc., the traditional terms of the sentence functions
(e.g. 'Object') appear as well (e.g. in Dukov, 1994).
Characteristics of Object (1)
(2)

3.1

meaning/interpretation = semantic roles


form = grammatical properties
(a) morphology
(b) SYNTAX

Semantic Roles of Objects

The most general semantic role related to the function of Object is Patient/Affected Object.
The Object semantic role, however, is most closely related to the meaning of the Verb and
it is therefore infinitely varied. The roles of some Objects can be otherwise typical for other
functions (Agentive, Possession, Instrument, Location).
(1)

3.2

(a)
(b)

Affected Object
Patient

He overturned THE CHAIR.


The mob killed JOHN BROWN.

(c)
(d)
(e)
(f)
(g)
(h)

Cause
Result of the action
Locative
Agentive
Possession
Instrument

I love JOHN
He wrote A BOOK.
John climbed MOUNT EVEREST.
The room dances TWENTY COUPLES easily.
We (have) got MANY LETTERS.
They threw STONES

Morphological Properties of Objects

Morphological CASE : Object Case ('Accusative')


is a structural Case, assigned by a Case assigner (V or P) to an adjoined NP.
English: - CASE-MARKING (only with central pronouns, often optional)
(2)

(a)
(b)

Mary loves Adam.


Adam loves Mary.

-->
-->

She loves *he/ him


He loves *she/ her.

(c)
(d)
(e)

Who/Whom does she love?


... the man who/whom she loves
Mary loves herself/*sheself

(3)

Object Case after Verbs and after Prepositions

(a)
(b)

I saw/introduced/met him/her/them.
We were talking about/with him/her/them.

personal pronouns
interrogatives
relative pronouns
reflexive pronouns

Object of the Verb


'Object' of Preposition

14

3.3

Syntactic Definition of Objects (Complements)

Objects are phrasal constituents, i.e. can be bare, but also very complex.
The category (part of sppech) related to Object function is Noun, i.e. NP (and its varieties)
and clause (semiclause).
Distribution: position with respect to (w.r.t.) other sentence members... see examples
(3)/(4)/(5) and above all in (6) on page 8 above, showing that Object immediately follows
the Verb. There is, however, another standard position of Objects: initial
- in WH questions (followed by Aux/Mod)
- in relative clauses, when Object is a relative pronoun (followed by Subject)
- in Topicalization structures (followed by Subject)
(4)

(5)

(a)
(b)

Whom did he love most?


What do you want?

(c)
(d)

(This is the men) who Mary loves most. OBJ - SUBJ - Mod/Aux - V
Syntax I hate.

OBJ - Mod/Aux - SUBJ

Summary
The canonical / standard / unmarked 'direct Object'
(1)

semantic role

A2, i.e. Patient/Affected Object/Theme/Goal...


(wide variety!!!)

(2)

(a) morphology

Object Case / ACC (if visible)

(b) syntax

Immediately follows V (but also initial)


usually NP (+ its proforms. Also PP, VP, clause)
Structural Object can be passivised

- The Summary above shows several criteria, not all of which are always atestable.
- English and Czech are typologically distinct (analytic vs. syntetic language) and therefore
distinct criteria may be accentuated.
(6)

English/Czech Definition of 'direct' Object

(a)

The direct Object in English is defined above all syntactically. = Object is the the
nominal complex which immediately follows the Verb, i.e. it is the POSITION
(= constituent order) that defines the structural Object.

(b)

The direct Object in Czech is defined above all morphologically = It depends on


CASE marking (direct Object is in Accusative)

There are consequencies of the distinct definition of Object in Czech and English. Some
will be mentioned discussing the process of passivization in Chapter 5 on page 23 below.

15

3.4

Exercises

(7)

EXERCISE ===========================================
Which kind of semantic roles do the Objects in the sentence below represent?

Bonnie runs THE RACE / A MILE / A SHOP.


(8)

EXERCISE ===========================================

Looking at the examples below, state/explain, which Case marking is obligatory in


English and which is optional.
(a)
(b)
(c)
(d)
(e)

Who/*whom wants to go first?


Who/%whom do you like most?
About whom/%who are you talking all the evening?
Who/%whom are you talking about all the evening?
*Who/whose sister did Benjamin marry?

(9)

EXERCISE ===========================================

In the following examples, underline the Verbs and discuss the category / constituent
of the Objects (mark the left border bracket with a categorial label as in (a)).
(a)
(b)
(c)
(d)
(e)
(f)
(g)
(h)
(i)
(j)
(k)
(l)
(m)

He loves [NP the most beautiful girl with long hair.


He loves Julie/her.
They shaved themselves/each other.
I prefer typing my articles.
He stopped smoking.
I don't like playing cards.
He buys what he wants.
He says he doesn't mind.
We would appreciate (it) if you would leave.
The situation requires that everyone should do his best.
Which of those handsome boys does Julie love most?
How many new black coats do you want to buy?
Only huge spiders with black hairs I hate more than syntax..

(10) EXERCISE ===========================================


Find direct Objects in the following sentences. Justify your choice, i.e. show that the
Objects have the properties of direct Object as in (5) on page 15 above, or that they
represent an acceptable variety. Try to give as many properties as possible (e.g. use
substitution to show the Case or part of speech).
(a)
(b)
(c)
(d)
(e)
(f)

Emily saw me.


All the people have the right to freedom.
My brother really loves watching Friends.
Whom did you invite for dinner.
I cannot say that Julie buys many things.
I cannot say what Julie bought.

16

(g)
(h)
(i)
(j)

Julie wrote Bill a letter.


Julie wrote a letter to Bill.
Those new rules I cannot understand.
Adam likes to read novels by Jane Austen.

(11) EXERCISE ===========================================


(i)

State/repeat a descriptively adequate generalization for the order of verbal


Complements. Use examples, compare with Czech.

(ii)

In the following examples mark the ungrammatical sentences *


If they are ungrammatical, explain the reason for their ungrammaticality.

(a)
(b)
(c)
(d)
(e)
(f)

I saw yesterday Emily.


I saw him yesterday in the garden.
I saw him in the garden yesterday.
I visited Emily yesterday.
I visited yesterday Emily..
I visited yesterday my older brother who moved to Prague in 1997.

(12) EXERCISE ===========================================


Explain the order of The constituents in the following sentences referring to the Heavy
constituent shift (=the 'heavy' (=complex, long) constituents tend to be peripheral).
(a)
(b)
(c)
(d)

Peter gave the book to Julie, not to her brother William.


Peter gave only the book his father brought from Chicago to Julie.
Peter gave Julie the book his father brought from Chicago
Peter gave Julie not her brother William the book.

(13) EXERCISE ===========================================


The following examples contain so called phrasal Verbs. Some of the sentences are
acceptable, others are ungrammatical. Mark the ungrammatical * and explain the reason
for the ungrammaticality.
Find a descriptively adequate generalization for the distribution of verbal particles.
(a)
(a')
(b)
(b')
(c)
(c')

Joe gave back the book to Elisabeth.


Joe gave back Elisabeth the book.
Joe gave the book back to Elisabeth.
Joe gave Elisabeth back the book.
Joe gave the book to Elisabeth back.
Joe gave Elisabeth the book back.

(d)
(d')
(e)
(e')
(f)
(f')

I sent away him the car.


I sent away the car to him.
I sent him away the car.
I sent the car away to him.
I sent him the car away.
I sent the car to him away.

17

PASSIVIZATION

H&P (2002) pp. 1427-1441 ; QGLS (2004) pp.159-171;Dukov (1994) pp. 249-272;
H&P (2005) pp.217-222;
Passivisation is a process typical for structural (direct) Objects. Elements which can
passivize are syntactic Objects of the Verb (in English they can also follow the
Preposition).
(1)

(2)
JOHN
Subject
Agent

(a)
(b)

Benjamin wrote/saw/bought/discussed some books.


The books were written/seen/bought/discussed by Benjamin.

(c)
(d)

Our family was looking forward to Mary.


Mary was looked forward to by our family.

The Process of Passivization: what happens?


WROTE
Predicate

a BOOK
Object
Patient

the BOOK
Subject
Patient

was
WRITT-EN
Predicate

by JOHN
PP Object
Agent

(a)
(b)

the Verb changed its form. (wrotewas written, morpheme be+-en)


Agent and Patient are distributed in a distinct way, they acquire distinct sentence
functions. (NOT **Subject and Object exchanged places.)
See sections2.1.1 and 2.1.2 on page 4.

(c)

as a result of (b), the Arguments changed the formal properties


(= Case, Prepositions, in English also position etc.)
(John/he by John/him, some books/them the books/they)

4.1

Function (Pragmatics/Usage) of Passivization

Results of passivization: semantic/pragmatic + formal. Passivization changes the formal


realization of the semantic valency (especially the external top semantic role of Agent).
It results in either
a)
demoting the Agent (deagentization), or
or
b)
rhematization of the Agent.
4.1.1 Demoting the Subject/Agent of the active Verb = Deagentization
(3)

The book was written.

(i) Patient becomes Subject/=Theme,


(ii) Predicate can become clause final/=Rheme
(iii) Agent disappers

(4)

Agent is

(a)
(b)
(c)
(d)
(e)

(i)

general

(ii)

author

(iii) hidden

It is believed...
It can't be explained.
As has been stated before...
This was studied in detail.
The team was beaten.

18

(iv) none

(f)
(g)
(h)

The house was searched.


The city is situated...
The two forms are distributed equally.

In Czech, Subjects can be partially demoted by dropping, but the deagentization is not full
and therefore this kind of passivization appears, too (especially to rhematize/focus
instrument or some Adverbial).
(5)

(a)
(b)

Petr/on napsal knihu. Napsal knihu.


? Kniha byla napsna.
Kniha byla napsna na zelenm pape/skvle/v 15. stolet.

4.1.2 Focusing the Agent which becomes 'by-' Adjunct


(6)

The book was written by Peter.

(i) Patient becomes Subject/=Theme,


(ii) Predicate remains neutral
(iii) Agent is clause final/=Rheme

The realization of the Subject of the active sentence depends also on its semantic role. True
Agents are canonically realised with the Preposition by.
(7)

(a)
(b)
(c)
(d)

The phenomena was first demonstrated by John Brown


I was impressed by his discipline
The metal was flattened by the machine
The door was opened by/with a key.

(8)

(a)
(b)

John opened the door with a key.


The key openes the door easily.

(9)

Inanimate Subjects can have also other Prepositions.


(a)
(b)
(c)
(d)

Sculpture interests her


His attitude surprises me
This prospect delights us
It worries me

(=Agent)
(=Agent?)
(=Instrument?)

She is interested in/by sculpture


I am surprised at his atitude
We are delighted at the prospect
I am worried about it

In Czech, Subject can be rhematized by the change of word order, which seems
'simpler/easier' than passive transformation. Therefore this kind of passive structures are not
used often, unless stylistic reasons make them more plausible (especially with very long
and complex Subjects).
(10) Compare the availability of the structures below (which constituent is in the
rhematic/final position).
(a)
(b)
(c)

House provedl operaci.


Operaci provedl House.
?Operace byla provedena Housem.

(d)

Pacient nepeil, pestoe operace byla provedena tmi opravdu nejlepmi chirurgy
naeho oddlen za asti genilnho doktora Hause a jeho t ambiciznch
asistent.

(a')
(b')
(c')

House did the operation.


*The operation did House.
The operation was done by House.

19

4.2

Restrictions on passivization

Passivization is a very productive, general and universal transformation, still, not all
V-Objects can be passivised. Some cannot be passivised because of formal reasons, other
beacuse of some semantic restriction, some are simply idiosyncratic.
4.2.1 Categorial incompatibility
recall that passivization targets direct Objects, and those are cannonically NPs.
Object, however, can be expressed with other constituents as well, some of which do not
tolerate passivization. See below
(11) *Reflexive pronouns/*Infinitives/?Gerunds/?Clauses
(a)
(b)
(c)
(d)

He excused himself.
They hate each other.
Children love to read.
He admitted reading it.

(e)

He suggested/shouted/whispered that such a case exists.

*He was excused by himself.


*They are hated by each other.
* To read is loved.
* Reading it was admitted.

??That such a case exists was suggested/shouted/whispered.


It was suggested/shouted/whispered that such a case exists.
(expletive it + clausal associate)

4.2.2 Semantic Restrictions


Some Verbs allow passivization which removes the Subject of the active counterpart, but
they do not allow the by-phrase, because of the semantic role of the Subject.
The by-phrase is always OK with Agents, appears also with Theme+Agent or Goal, but
the by-phrase cannot be a pure Theme.
(12) Agent (= active participant) vs. Theme role
(a)
(b)
(c)
(d)

A dog crossed the road.


The fire crossed the road.
He received a good education.
He suffered a heavy blow.

The road was crossed by a dog.


*The road was crossed by the fire.
*A good education was received by him.
*A heavy blow was suffered by him.

Some sematic factors can disallow the pasivization completely, though it is not very clear
which, how and why. (Object, to passivise, 'requires to be affected' by the transitive Verb).
(13) some Verbs meaning 'possession' do not passivise, but not all (and not always)
(a)
(b)

(c)

Bill possesses a book.


The book is possessed by Bill.
Bill has a book.
* The book is had by Bill.
(transitive Copula)
Bill got a book.
* The book was got by Bill. (get = was given?)

20

BUT (d)
(e)

There was nothing to be had.


There was nothing to be got.

(14) 'transitive Copulas':


(a)
(b)

4.3

to cost = the price is...,


to measure = the measure is... etc

The book costs 5 krowns.


*5 krowns was cost (by the book)
The bridge measures 1 mile.
*1 mile was measured by the brodge.
A Remark about Semantic notions of Action versus State

There is a high level of similarity between the passive participle (a Verb following the Aux
be) and an adjective derived from V (which can follow the Copula be). In Czech the
distinction can be seen in morphology n/n.
(15) (a)
(b)
(b)
(c)
(d)

Don Corleone nechal zavradit rozhodujcho svdka.


Rozhodujc svdek byl zavradn /zavradn.
Zavradn/*Zavradn svdek nemohl vypovdat.

... Verb
...Verb / Adjective
... Adjective / * Verb

Dm byl postaven/-n z cihel minul rok firmou Svanska.


Petr je u unaven/-n.

In English, the passive participle morpheme ed is similar/identical to? the derivational


morpheme ed, which creates derived adjectives. Both of the forms can appear after the
Verb be and can be mistaken and the term thus seems categorially 'fuzzy'.
Notice distinctions between the two.
(16) The distinction between
adjectival passive (=Verb) and adjectives (=APs following copulas)
Verbal passive is characterised by:

Adjective is characterised by :

(a)

(a')

(b)
(c)

activity reading
(i.e. can be perceived as progressive)
ability to contain by- phrase
active counterpart

(b')
(c')
(d')

stative reading
(when progressive, than temporary)
no by-phrase
concrete modification
gradability

(17) Process vs. temporary state


The scale from state to activity is gradual and some forms, if bare, remain ambiguous.
(a)
(b)

the house is being built


John is being silly /tired

process (=V)
temporary state (=Adj)

(18) Marked non-stative (be/get Auxiliary)


(a)
(b)

we are being served


she is being blamed

(a')
(b')

we got served
she got blamed

21

(c)
(d)

the matter is being attended to


such things are being noticed

such things get noticed


she never got caught
it doesn't get solved by being talked about.

Modification, by-phrase

(19)
(a)
(b)
(c)

(d')
(e')
(f')

This chicken was kill-ed by Harriet.


This chicken is freshly kill-ed (*by Harriet).
A freshly kill-ed (*by Harriet) chicken (*by Hariet).

(20) Gradability
(a)
(b)
(c)

I am very/rather/more tired/exhausted/surprised/irritated
this very/rather/more is unexpected/unjustified/clear-cut/widespread
the car is *very/*rather/*more repaired

4.4

Exercises

(21) EXERCISE ===========================================


(i)
(ii)

In the following couples of sentences find elements expressing


sentence functions (i.e. Subject - V - Object ) and
semantic roles (i.e. Agent - Verb - Patient)

(a)

Everybody saw Emily.


Emily was seen (by everybody).

(b)

The hunter killed a rabbit.


The rabbit was killed (by a hunter).

(c)

Mary feeds her dog.


The dog is fed (by Mary.)

(22) EXERCISE ===========================================


Discuss the underlined constituent in terms of its
(i) semantic role (wrt the verb),
(ii) sentence function ,
(iii) sentence dynamism, i.e. Topic/Theme and Focus/Rheme.
(a)
(b)
(c)

Petr Novk postavil dm.


Ten dm postavil Petr Novk.
Ten dm postavil Petr Novk.

(d)
(e)
(fi)
(g)

Peter built a new house last year


The new house was built by Peter last year
Peter built a house last year.
The house was built by Peter last year.

22

(h)
(i)
(j)
(k)

Vichni musej dokonit sv referty do konce prosince.


Referty musej bt dokoneny do konce prosince.

Everybody must finish their papers by the end of December.


The papers must be finished by the end of December.

(23) EXERCISE ===========================================


(i) Compare the following couple of sentences wrt to which expressions carry the sentence
functions of Subject - V - Object ) and the semantic roles (i.e. Agent - Verb - Patient)
(a)
(b)

I haven't seen Peter.


Peter I haven't seen.

(ii) Try to find out what the distinction between (a) and (b) above is.
(Describe the so called "Topicalisation" (of object). How does the form changes? What are
the pragmatic consequeneces? Give more examples.)

COMPARING CZECH AND ENGLISH PASSIVES

H&P (2002) pp. 14271441 QGLS (2004) pp.159-171;Dukov (1994) pp. 249-272;
H&P (2005) pp.217-222;
The process of passivisation in English and Czech seems very similar, though some
distinctions w.r.t. frequency and usage were already mentioned in (and above) (5) and
(10) on page 19. The following discussion concentrates on the formal distinctions.
(1)

(a)
(b)

Jan pedstavil Marii.


John introduced Mary.

Marie byla pedstavena (Janem)


Mary was introduced by John.

In both English and Czech the passivization targets the 'direct Object'.
(2)
John

5.1

/ HE

Mary / SHE

introduced

Mary / HER

was introduced

(by John/him).

The Double Object Structures

Recall the definition of Object in section 3.3, esp. (6) on page 15.
Following the definition (6) on page 15, the Czech sentences (a/b) in (3) have identical
direct Object, since the same NP is in Accusative. Assuming Object in English is the NP
immediately following the Verb, there are distinct candidates for Objecthood in the
English translation (c/d).
(3)

(a)
(b)

Petr dal MariiDAT knihuACC.


Petr dal knihuACC MariiDAT .

CASE (ACC) is crucial for Czech Object

23

(c)
(d)

Peter gave [NP a book] to Mary.


Peter gave [NP Mary] a book

POSITION counts most for English Object

The claim is supported by the process of passivisation: Structural direct Objects can be
passivised. In English the candidates are both the NP following the Verb, in Czech it is only
the NP in Accusative (DAT cannot passivise)
(4)

(a)
(b)

Peter wrote a letter to John.


Peter wrote John a letter.

A letter was sent to John.


John was sent a letter.

(c)
(d)

Petr napsal dopis Janovi


Petr napsal Janovi dopis.

Dopis byl napsn... /*Jan byl napsn...


Dopis byl napsn... /*Jan byl napsn...

The process of passivisation in the double Object structures (see (3)/(4) above)
can be described as follows:
(5)

The so called 'dative' PP in English, which can also be expressed as NP


immediately following the Verb, can be passivised.

English double Object structures are complex phenomena and remain theoretically
chalenging, but they cannot be explained by their Czech translation (not all English
speakers speak Czech). To perceive the distinction between English and Czech in a way
suggested in (5) is superior to saying "in English Dative Object can passivise", because
(i) English has no Dative morphology,
(ii) not all Czech Datives can passivise in English (Chce se mi spt).
I. Not all English 'double Objects' have the alternation [NP,PP] [NP,NP] . The so called
'datives' with Preposition to are most likely, however some for 'datives' are also possible.
(6)

(a)
(b)

Carolyn baked that cake for my children.


Carolyn baked my children that cake.

... NP, for PP


... NP, NP

II. Most of the structures which allow alternation, [NP,NP] vs. [NP,PP] have also 2 passive
structures, but not all. The Preposition to/for seems to make the distinction.
(7)

(a)
(b)

They gave a book to me.


They gave me a book.

A book was given me.


I was given a book.

(c)
(d)

They bought a hat for me.


They bought me a hat.

.A hat was bought for me.


*I was bought a hat

Some 'direct Object' passive structures are also marginally acceptable without the
Preposition to.
(8)

(a)
(b)
(c)
(d)

They offered a woman the job


They offered the job to a woman
They fixed the guests a lunch
They fixed a lunch to the guests

A woman was offered the job.


?The job was offered a woman.
The guests were finally fixed a lunch.
?A lunch was finally fixed the guests.

24

5.2

Passivization with Verbal Complexes (Restructuralization)

Recall the definition of Object according to (6) on page 15: Direct Object in English is the
NP immediately following the Verb/Predicate. It follows that to recognise the Object, we
have to recognise the Verb, at least to be able to find the right edge of the Verb.
(9)

Subject -Aux, "Verb"

+ Object

In analytic language this structure need not be so transparent because of the presence of
Prepositions or some other 'words' between the Verb and the potential Object.
In the following example the verb 'look' is followed by a 'preposition' (Or is the preposition
'a part of ' the verb?)
(10) (a)
(b)
(c)

I look at John.
I lool forward to John.
I look for John.

(a')
(b')
(c')

Dvm se na Jana.
Tm se na Jana.
Hledm Jana.

(b)

He looked at the picture

5.2.1 Phrasal Verbs (Preposition Stranding)


(11) (a)

Pohldl na obrzekACC.

(12) (a)

(On)
He

pohldl
looked
VERB
I.

na
at
+

obrzek
the picture

PREPOSITION
II.

+ NOUN PHRASE
III.

In a sentence the Verb is often followed by a PP, i.e. a Preposition (and a NP). When we
accept binary system, the Preposition must be grouped with the Verb or with the NP.
The languages can differ w.r.t. the posibility of grouping of (some) Prepositions with
another element.
(A)

I.

(B)

either

I. + II.

(C)

or

I.

II.

III.

III.
II. + III.

Assuming the example in (A/a), in (B/b) the grouping is V + [P+NP], and in (C/c) it is
[V+P] + NP. The (C/c) grouping is very non-intuitive for Czechs, but typical for modern
English ("the preposition/particle becomes a part of the ('phrasal') verb")
(b)

On
He

pohldl
looks
VERB

na obrzek
at the picture
+

PREPOSITIONAL phrase

25

(c)

On
He

pohldl na
looks at
VERB+PREPOSITION +

obrzek
the picture
NOUN PHRASE = OBJECT

Assuming some Prepositions (particles) can count as a part of a complex Predicate


(compound Vs = V+P), the following nominal structure is Object = can be passivised and
questioned as NP. In these structures the Preposition remains related to the Verb (it
becomes so called stranded Preposition).
(13) Contexts for Preposition stranding: (i) WH questions and (ii) passivization.
(a)

Mary is looking at [those flowers] very carefully.

(b)
(c)

What is she looking at? ( % At what is she looking?)


Those flowers were looked at very carefully.

Not all [V+P] combinations can be 'restructured' in English, i.e. not all Ps can be stranded.
- Some [V+P] combinations are synchronically unified (so called phrasal Verbs),
- some [V+P] combinations can be optionally restructured (V+adjacent/Complement PP),
- some [V+P] combinations cannot ever be restructured (V+nonadjacent/Adjunct PP).
(14) (a)
(b)

Who were you looking for?


Who were you looking at?

(15) (a)
(b)
(c)

You are looking at the picture with Adam.


* Who were you looking at the picture with?
With whom were you looking at the picture?

??For whom were you looking?


At whom were you looking?

5.2.2 Verbo-nominal phrases : complex Predicates


In English, also some [weak Verb+Noun] combination can be perceived as one unit: They
are semantically opaque expressions, i.e. they create a "semantic unit" and they are also
formally unified. (See also 5.2.2.1 on page 27.) Such complex Verbo-nominal Predicates
can be followed by a PP.
(16) (a)
(b)

take advantage [PP of something


put an end [PP to something

(17)
People

took
VERB

ADVANTAGE
+ OBJECT

of his mistake.
PP (= P + NP)

'advantage' follows the Verb take, therefore it is its Object and can be passivized.
PASS

ADVANTAGE was taken of his mistake.

26

(18)
took

Liz

advantage of

HIS MISTAKE.

'weak' VERB + NOUN + P


= 'complex Verb' (verbo-nominal complex)

OBJECT

'his mistakes' follows the complex Verb take an advantage of , therefore it is its Object
His mistake was taken advantage of.

PASS

The verbonominal complex 'take advantage of' allows the enalysis for two direct Objects.
We can prove it using the existence of (i) two passive structures and also (ii) two WH
questions for Object.
(19) (a)
(b)
5.2.2.1

What can be taken of his mistake?


What can we take an advantage of?

An advantage.
His mistakes.

V+N Compounds

In English, we must distinguish Objects from the 'Object-like' parts of a complex verbal
form. Appart from the verbonominal complexes, English has also Verbal compounds in the
form of V + N.
(20) (a)
(b)
(c)
(d)
(e)

I had a walk / a nap.


He made a mistake / a bet.
They took trouble / a walk.
The meeting took place.
He took courage.

The V+N compounds are semantically opaque expressions, i.e. they create a "semantic
unit" and they are also formally unified.
The process of compounding (V+N) is, however, subject to diachronic change. When the
process is finished, the compound become inert w.r.t. syntactic operations: compounds do
not allow passivization (or questioning) of the items which may look like direct Object.
Compare the properties of the Verb + Noun (Phrase) complex w.r.t. two properties:
(i) questioning and (ii) passivization of the N(P), which show to which extend is the N(P)
independent on the Verb and to which extend they are compounded.
(21) WH question on Object
He took a book/a nap/courage. - What did he take?
(a)
(b)
(c)
(22)
(a)
(b)
(c)

- A book.
- % A nap.
- * (A) Courage.
Passivisation

The book was taken (by Benjamin).


% The nap was taken (by Benjamin).
* (A) Courage was taken (by Benjamin).

27

(23) (a)
(b)
(c)
5.3

take a book
take a nap
take courage

always V + NP Object
% marginal [V+NP] compound
only [V+NP] compound

Mediopassive

Similarity of the 'reflexive passive' (Cz) and 'mediopassive' (En):


Active form (there is no passive morpheme: be+V-en), but the meaning is the similar to
passive (Subject is interpreted as Patient and Agent is unexpressed)
(24) (a)
(b)

Dvee byly zaveny.


The door was closed.

(a')
(b')

Dvee se zavely.
The door closed.

the use of Mediopassive


(25) general characteristics
(a)
(b)

the clothes wash easily


that is easy to believe

=it is easy to wash the clothes


=it is easy to believe a thing like that

(26) modal possibility


(a)
(b)
(c)

the lid shuts


the door locks
his poem does not translate well

=The lid can be shut


=The door can be locked
=His poem cannot be translated well

(27) individual characteristics


(g)
(h)
(i)
5.4

he counts among the best


the text sings to the tune...
the dress buttons down the back

=he is/can be counted among the best


=the text is/can be sung to the tune...
=the dress is/can be buttoned ...

Exercises

(28) EXERCISE ===========================================


Explain which sentence on the right is the equivalent of the example on the left
(i) considering the sentence functions of Subject and Object,
(ii) considering the Topic/Theme and Focus/Rheme interpretation,
(iii) considering the semantic roles of Agent and Patient.
(a)

The letter was written by your brother.

(i)
(ii)

Tvj bratr napsal dopis.


Dopis napsal tvj bratr.

(b)

Peter was seen by Mary.

(i)
(ii)

Marie vidlaPetra
Petra vidla Marie.

(c)

Krlka zabila babika.

(i) Granmother killed the rabbit.


(ii) The rabbit was killed by Grandmother.

28

(29) EXERCISE ===========================================


Find and underline the Objects in the following examples. Respect the definitions in
(6) on page 15, i.e. recall that in English (because of the word order) distinct elements can
count as direct Objects than in Czech. Check your analysis by passivization.
(a)
(b)

Henry sang a song to Mary.


Henry sang Mary a song.

(a') Jindich zpval psniku Marii.


(b') Jindich zpval Marii psniku.

(c)
(d)

Jude sent a parcel to her brother.


Jude sent her brother a parcel.

(c') Jitka poslala balk svmu bratrovi.


(d') Jitka poslala svmu bratrovi balk.

(30) EXERCISE ===========================================


Passivise the underlined 'Objects' and watch what happens with the capitalised
Prepositions. Can you translate the Verbs to Czech? What are their Objects? Can you
passivise them?
(a)

He blamed [the teacher][FOR his failure].

(b)

PASS: ......................................................................................................................
He blamed [his failure][ON his teacher].

(c)

PASS: ......................................................................................................................
Adam entrusted [a task] [TO Mary].

(d)

PASS: ......................................................................................................................
Adam entrusted [Mary] [WITH this task].
PASS: ......................................................................................................................

(31) EXERCISE ===========================================


In the following sentences underline the Verb. Is there a clear unambiguous "NP
following the Verb" which would be a candidate for Object? Mark such NP with a bracket
[NP . Recall that such NP would be able to passivise and check your analysis by
passivization. E.g. Mary saw [NP John. John was seen (by Mary).
(a)
(b)
(c)
(d)
(e)
(f)
(g)
(h)
(i)

Jessica writes articles for the newspaper.


William will be helping you tomorrow.
Mary was not allowed to go to the cinema yesterday.
His brother is looking for a girlfriend.
Marilyn will take advantage of her father's money.
He was looking at the picture.
I have been looking forward to your visit.
He sleeps in his bed.
Jane has got a new friend.

(32) EXERCISE ===========================================


Consider the following sentences none of which has an overt element representing
Agent. Which of the passive sentences still contains a semantic notion of Agent, i.e. which
of the passive examples allow a question 'WHO did it?'

29

(a)
PASS (b)
PASS? (c)

Petr zavel dvee.


Dvee byly zaveny.
Dvee se zavely.

Peter closed the door.


The door was closed.
The door closed.

(33) EXERCISE ===========================================


Make pasisve structures from the active sentences below and describe changes with
respect to (a) semantic roles, (b) grammatical functions, (c) pragmatic roles. Mention also
possible reanalysis and comment on possible Verbo-nominal complexes.
(a)
(b)
(c)

The boy gave that article to Mary.


PASS:
The boy gave Mary the article.
PASS:
The terrorist attack put an end to the hope of a better future.

(d)

PASS:... .
The terrorist attack put an end to the hope of a better future.

(e)
(f)
(g)
(h)

PASS:.. .
Theodor sent the paper to his teacher. PASS:
Theodor sent his teacher the paper.
PASS:
Mary looked forward to Bill.
PASS:
PASS:
James slept in that bed.

(34) EXERCISE ===========================================


Compare the following examples (use your own examples, too) and answer the following
questions:
(i) How does passivisation influence the distribution of Semantic roles of the Verb?
(ii) What is the function of passivisation w.r.t. the sentence dynamism?
(iii) Mention the distinctions with Czech.
(i)

(a)
(b)
(c)

William gave the bunch of flowers to his mother.


The bunch of flowers was given to his mother.
His mother was given a bunch of flowers.

(ii)

(a)
(b)
(c)

Emma wrote the book very well.


The book was written very well.
The book writes very well.

(35) EXERCISE ===========================================


Mark the level of (un)grammaticality of the sentences (* for ungrammatical, ? for
formally marked) and explain/justify your evaluation.
(a)
(b)
(c)
(d)
(e)
(f)
(g)
(h)

The exam was taken by all the students.


The courage was taken by all the studnets.
The book was recommended by our teacher.
To read was recommended by our teacher.
Peter was spoken about.
Peter was spoken with Mary about.
The letter was written by a pencil.
The road was interrupted by a river.

30

SUBJECT

H&P (2002) pp. 235-243, QGLS (2004) pp. 724-767; pp.755-766;


Dukov (1994) pp. 390-422; H&P (2005) pp.12-13,67-70;
Subject is a main sentence member (sentence function). It can be defined w.r.t. its
(a) semantics (interpretation) and
(b) form (i) morphology and (ii) syntax (=distribution, special operations).
6.1

Semantic Roles of Subjects

Quirk/Greenbaum/Leech/Svartvik (2004) pp. 740-754;


Semantic vs. Syntactic Subcategorization
See also 'valency' in section 1.1.1 on page 1 and section 2.1.1 on page 4.

(1)

'hit' is a VERB and its semantic


selection contains A1=Agent and
A2=Patient.
In syntax, 'hit' must be complemented
by a nominal element (Object), which
canonically follows the verb.

hit: V, < Agent, Patient >


V, [-- NP]

Notice that semantic subcategorization < Agent, Patient > mentions two semantic
arguments: (i) A1, Agent (i.e. usually Subject) and (ii) A2, Patient (i.e. usually Object). The
syntactic subcategorization V, [-- NP] refers to the right hand complementation only. It is
because only that element can be specific w.r.t. the Verb. The form of Subjects depends on
the type of the sentence, not on the Verb.
(2)

(a)

[VP To read letters ] is irritating.

(b)

I do not like [VP to go home soon].

The function of Subject standardly realises the (highest) semantic role of Agent.
(3)

(a)

Mary heard George to hit the window with stones.

(b)

MARY
heard GEORGE/the WIND hit the WINDOW (with STONES).
Experiencer
Agent /Force
Patient/Goal
(Instrument)

Consider the need of conscious activity for the notion of Agent.


Non-active (passive) Agent is often called Experiencer, unconscious Agent is a Force.
BUT: Are all Subjects Agents? - No, in Subject position also non-Agent can appear.
(4)

(a)
(b)
(c)
(d)
(e)

JOHN [Agent] opened the door.


MARY [Experiencer] can hear the noise.
The HURRICANE [Force] took away many roofs in the neighbourhood.
THE KEY [Instrument] opened the door.
THE DOOR [Theme/Patient] opened/broke.

Are all Agents Subjects? - No, Agents can be realized as non-Subjects.


(5)

The letter was written by Bill.

31

Is the role of the Subject the same in different languages?


- No, especially with Verbs with ambiguous arguments, the realization varies.
(6)

(a)
(b)

Peter really likes the book.


Petrovi se ta kniha opravdu lb.

Do all Subjects have some/any meaning/Semantic role?


- No, some Subjects are purely formal (expletives).
(7)

(a)
(b)

Pr. / ?? To pr. / ?? Ono pr.


It rains.
/ *Rains.

(8)

(a)
(b)
(c)
(d)
(e)

It is raining/snowing.
It is late.
It seems that John arrived late.
It is outrageous that nobody helped you.
It is easy for me to go.

weather Verbs (NO Agent)


linking/anticipatory it

Conclusion: The examples above showed that Subject is a fromal concept not directly
related to the interpretation. Subject is a necessary part of each sentence, although
(i) the semantic role of the function of Subject can vary cross-language,
(ii) the semantic role is neither necessary nor sufficient for the function of Subject,
6.2

Morphological Properties of Subjects

The formal characteristics (morphologic and syntactic) are similar in both Czech and
English, but the hierarchy between them is distinct
Czech

1.
2.

Case/Agreement
Word order

English:

(a)

- CASE-MARKING (with personal pronouns):


special/distinguishable/regular/frequent/obligatory form

(b)

- AGREEMENT (of the Verb-PRES and its Subject-3sg)

(a)
(b)

He(NOM) see-s(AGR) her(ACC).


*Him sees her. *He see her.

(9)

English

1.
2.

Word order
Case/Agreement

6.2.1 Subject Case in English (Nominative)


BUT: Nominative/Subject Case in English is highly marked. It does not appear in
separation and in coordinated structurs.
(10)

(a)
(b)
(c)
(d)

Who did it? - Me.


William and him are good students.
It was I/me that Mary brought the present.
Nobody but he/him can help you.

32

In non-2nd person imperatives and with infinitives the Nominative does not appear at all.
(11)

(a)
(b)
(c)

Let's go home, shall we?


Let him help you.
Let them do what they want.

English Agents/Subjects? in nominal - infinitive constructions:


(12)

(a)
(b)
(c)

For him to go home seems very complex.


After his/*he/*him saying goodbye, I left.
His reading of the article is really irritating.

6.2.2 Subject Predicate Agreement


Also AGREEMENT in Person/Number can be idiosyncratic.
The distnction between so called 'semantic' vs. 'formal' agreement appears
(a) with Copula, when Subject and nominal Predicate have distinct features
(b) when Subject NP is 'complex': is coordinated, contains quantifier, of-phrase, etc.
(the head of the Subject is not unambigous).
(13)

(a)
(b)
(c)
(d)
(e)
(f)
(g)
(h)
(i)
(j)
(k)

His only success was his short stories.


His short stories were his only success.
What we need most is/are sufficient funds.
Two years is a long time to wait.
Bread and butter is a nice breakfast.
A large number of students are granted scholarships.
Either you or he is mistaken.
Either he or you are mistaken.
A book or flowers are a good present.
Flowers or a book is a good present.
The police is/are looking for the criminal.

6.2.3 Pro-Drop Languages


Languages with rich verbal inflection for Agreement (Czech but not English) can leave the
position of the Subject empty. A Subject 'pronoun' is dropped and the Subject is realized
only as bound morpheme on the Predicate. Czech is a pro-drop language, English is not.
(14)

(a)
(b)
(c)
(d)

Odeel jsem. (=1sgM)


Pomhali nm cel den. (=3pl)
Pr. (=3sgN)
Pomoz si sm! (=2sg)

(a')
(b')
(c')
(d')

I left.
They were helping...
It rains.
Help yourself, will you!

Realization of the grammaticalised feature in the form of free or bound morpheme.


Synthetic languages prefer the bound form, analytic the free one. In some cases the bound
and free form co-occur.
bound
/
free
more beautiful
..... -er /
more
(15)
(a) Nicer
(b) vykldat
to talk
..... -t
/
to
(c) (my) chodme
we go / he reads
..... -me /
we

33

There are only few contexts in English with no overt Subject.


(16)

6.3

(a)
(b)

Idiosyncratic
Imperative:

(c)

Diary style:

Thank you.
(i) Open the window, will you? Help yourself!
(ii) Let's go, shall we?
Came back soon. Made a dinner and watched TV....

Pragmatic Roles of Subjects (related to linearity)

Recall the pragmatic Aspects of the discourse which are related to the distinction between
"old" and "new" information and to the stress the speaker wants to make. Those are often
expressed with a constituent-order (i.e. linearity). The position of Subject is often initial,
and therefore Subjects have a strong tendency to be the "Topic/Theme" of the proposition.
(17) Discourse order, sentence dynamism, Functional Sentence Perspective
THEME / OLD /TOPIC
SUBJECT

VERB RHEME / NEW / FOCUS


VERB
OBJECT

In English (because of its constituent-order is grammaticalised) the correlation between


pragmatic factors and word-order is not as strong as in Czech, but it still exists.
(18) WH questions ask for the Rheme/Focus.
WHO saw Mary? (a) John saw Mary.
(b) = * Mary saw John.
(c) = Mary was seen by John.
(19)

English: SVO / * OV-S

(a')
(b')
(c')

?? Jan vidl Marii.


Marii vidl Jan.
Marie byla vidna (??Janem).

Czech: SVO / OV-S

There is a preference in all languages to identify Subject with the Agent and the Topic but
this tendency is weaker in English than in Czech. There is also a strong tendency to repeat
parallel structures, when Subjects "tend" to remain Subjects unless marked otherwise).
(20)
6.4

Peter saw Paul and he(=PETER) greeted him(=PAUL).


Exercises

(21) EXERCISE ===========================================


Write down the syntactic and semantic subcategorisation of the following Verbs.
(See (4) on page 4)
(a)
(b)
(c)
(d)

catch
say
have
believe

.......................................................................................................................
.......................................................................................................................
.......................................................................................................................
.......................................................................................................................

34

(e)
(f)
(g)
(h)
(i)

read
bear
appear
sell
seem

.......................................................................................................................
.......................................................................................................................
.......................................................................................................................
.......................................................................................................................
.......................................................................................................................

(22) EXERCISE ===========================================


Which semantic roles are typical for Subjects? State the semantic role of Subject in
the following sentences.
(a)
(b)
(c)
(d)
(e)
(f)
(g)
(h)
(i)

Jude
[
The key
[
Martin
[
Hannah
[
The stone [
The enemy [
Benjamin [
The book [
Debora
[

]
]
]
]
]
]
]
]
]

wrote a letter to Bill.


opened the door.
saw Mary in the park.
was hit by Susan/the stone.
hit the roof.
was destroyed.
heard the thunder.
sells well.
was given a bunch of roses.

(23) EXERCISE ===========================================


Find the morphemes realizing the Subject:
(a)
(b)
(c)

Moc jste mi nepomohla!


Stmv se.
Thank you ever so much!

(d)
(e)
(f)

Let's go!
Open the window, I beg you!
Went back. Met Jessica.

(24) EXERCISE ===========================================


What is Subject/Object in the following sentences? Give arguments for your claim.
Explain the ungrammatical example.
(a)
(c)

Let us go home, shall we.


Let us go home, will you.

(b)
(d)

Let's go home, shall we.


*Let's go home, will you.

(25) EXERCISE ===========================================


What is the Subject in the following sentences? Give arguments for your claim
(mention the semantic, morphological and syntactic criteria).
(a)
(b)
(c)
(d)

It is impossible Let us go home, shall we.


Let's go home, shall we.
Let us go home, will you.
*Let's go home, will you.

(26) EXERCISE ===========================================


Consider who is a preferred Subject of the second coordinate.
(a)
(b)

Bill udeil Johna a pak odeel. (On)


Bill udeil Johna a on/ten pak odeel.

on =
on/ten =

.................................
.................................

35

(c)
(d)

Bill hit John and then ??/he left.


Bill hit John and then HE left.

(he) =
HE =

.................................
.................................

(27) EXERCISE ===========================================


Underline the Subjects. If there is an ambiguitychosethe most salient candidate. State
precisely which formal criteria you used to make your choice.
(a)
(c)
(e)

Chlapci vidli mue.


Kata honila husy.
Dvky vidly eny.

(b)
(d)
(f)

Vlky napadli mui.


eny vidla dvata.
IBM koupil Apple.

(28) EXERCISE ===========================================


Mark the acceptability of the agreement. Consider the criteria you apply. Give some
similar examples in English.
(a)
(b)
(c)
(d)
(e)

Shromdn studenti stli / stlo...


tyi studenti li / lo...
Do koly el/ la/ li Petr a Marie.
To je / jsem j.
Dti jsou nae radost.

SYNTACTIC PROPERTIES OF ENGLISH SUBJECTS

(a') Shromdn student stli / stlo...


(b') Pt/mnoho student li / lo...
(c') Petr a Marie el/ la/ li do koly.
(d') Bu ty anebo j budu / bude /budeme
(e') as jsou penze.

H&P (2002) pp. 235-243, QGLS (2004) pp. 724-767; pp.755-766;


Dukov (1994) pp. 390-422; H&P (2005) pp.12-13,67-70;
S

LINEARITY: Distribution, word order.


See the schemes in Section 1 on page 1.
XP
Which constituent can be the
XP in the scheme?
(1)

(a)
(b)
(c)
(d)
(e)
(f)
(g)
(h)
(i)

VP
V

YP

My older brother (who lives in Prague) is at home.


Adam / He / One never knows.
Putting it off won't make it any easier.
To leave so early would be impolite.
It's a pleasure to write for your journal.
Whoever stole my purse didn't get much.
That the ship was wrecked was feared.
It is feared that the ship was wrecked.
It is not determined WHO should be sent.

Subjects are prototypically


a)
b)

NPs or their substitutes (pronouns). They can be also


VPs (infinitives, gerunds) or

36

c)

finite clauses (with or without expletive it)

(2)

Subject position in main sentence patterns w.r.t. other sentence members:

(a)

Yesterday William did not go to school.


In the afternoom Mary cannot work...
Will Bonnie see his grandma tomorrow?
How many cars does Bill Gates have?
Who will help me?
Never will I do it!

(b)
(c)
(d)
(e)

. . . SUBJ - Mod/Aux - V ...


Mod/Aux - SUBJ - V ...
WH Mod/Aux - SUBJ - V ...
WH SUBJ Mod/Aux - V ...
NEG Mod/Aux - SUBJ - V ...

Syntactic Subject are signaled also throuh specific processes


(a)
(b)
(b)
(c)

Syntactic Subjects invert with the first Mod/Aux in questions


Short answers repeat pronominalised Subject.
Question tags reapeat pronominalised Subject
Syntactic anaphors (reflexives and reciprocals) take Subject/Agent for its antecedent.
Mary saw Adam.

(3)

(a)
(b)
(c)
(d)

(4)

Did Mary see Adam?


- Yes, she did.
Mary saw Adam, didn't she/*he.
Mary saw Adam herself/*himself.

Summary
The canonical / standard / unmarked 'Subject'
(1)
(2)

semantic roles
(a) morphology
(b) syntax

7.1

A1, i.s. Agent (some variety including none)


Subject Case / NOM (if visible)
Agreement on Predicate (3sg,Pr)
NP (+ its proforms. Also PP, VP, clause)
Immediately precedes the Predicate,
inverts in questions, appers in Question tags,
short answers...
It is an antecedent to anaphor

Dissociating the form and the meaning

7.1.1 Dissociated Subject (Expletives there and it)


English sentence must have an overt Subject. Assuming that we can define Subject w.r.t.
its (i) meaning, (ii) morphology and (iii) position, in some sentences there seem to be more
candidats for the role of Subject, one of which is expletive.
A.

Linking it

37

B.

Existential structures there is/there are...


Subject has two parts

(5)

(a)
(b)
(c)

(6)

(a)
(b)
(c)

(7)

It seemed obvious that Mary left, didn't it.


Did it seem obvious that Mary left?
That Mary left seemed obvious.
It seemed obvious.
It was not the best idea to go home, wasn't it.
Was it the best idea to go home?
To go to the cinema was not the best idea.
It was not the best idea.

Linking it (expletive) + associate Subject (semi-)clause


(i)
(ii)

the associate (semi-)clause follows the clause containing the expletive it


the expletive it is able to stay without the associate (in a salient context).

SUBJECTFFFOOORRRM
M
A
L
m
M
O
R
H
O
L
O
G
C
A
L
MA
AL
L - the rest of the clause ] [ SUBJECTSSSeeem
maaannntttiiiccc///M
MO
OR
RPPPH
HO
OL
LO
OG
GIIIC
CA
AL
L
Expletive
associate
In the following sentences the expletive is not it but there. There serves as a syntactic
Subject, while the associate NP controlls the verbal agreement and provides the semantic
content.
Subject has two parts
(8)

(9)

(a)
(b)
(c)
(d)
(e)

There is a book on the table, isnt there?.


Is there a book on the table.
*There is two books on the table.
* There is the books on the table.
* There is on the table.

violates Subject-Verb agreement


the associate must be indefinite
the associate is obligatory

Expletive there + associate Subject


(i)
(ii)

the associate (=indefinite NP) is in the same clause as the expletive there
the expletive there is not able to stay without the associate

SUBJECTFFFOOORRRM
M
A
L
m
M
O
R
H
O
L
O
G
C
A
L
MA
AL
L - Vcopula - SUBJECTSSSeeem
maaannntttiiiccc///M
MO
OR
RPPPH
HO
OL
LO
OG
GIIIC
CA
AL
L - LOCATION
Expletive
associate
7.1.2 Locatives / Directional structures
The existencial structures (See above in section 7.1.1.) devoleped from the
locative/directional structures. Recall that Old English had a free constituent order.
(1)

Locative PPs

(a)
(b)

In the house can be found a famous painting.


In the house is hanging a famous painting.

38

Directional PPs

(2)

(a)
(b)

Into the house ran Jessica.


Up came the number.

7.1.3 Raising to Subject


In English distribution (=the position) is the main signal of Subjecthood. The semantic role
of Subject (Agent) is not a necessary condition. Because of the loose correlation between
the semantic role and sentence function in English, the Subject of a finite Verb can realizes
also a semantic role related to another (infinitive) Verb. In Czech, the structures are rare.
(3)

(ono) Zd se, e Petr je unaven.

Petr se zd bt unaven.

It (=0) seems that Peter is tired.

Peter seems --- to be tired.

Raising to Subject: some argument (Agent or Patient) of infinitive rises to the position of
the formal Subject of a finite (agentless) Verb or 'impersonal' structure.

(4)

Peter seems to be reading a book. ....

Peter is

(i)
(ii)

Subject of seem
Agent of read

Consider the semantic roles of IT and JOHN and notice, how far can 'Raising' go.
(5)

(a)
(b)
(c)

It seems that John left home.


?? (That) John left home seems.
John seems to be at home.

(6)

(a)
(b)
(c)

It is impossible for John to come.


For John to come is impossible
John is impossible to come.

(7)

(a)

It seems that John was beten.


John seems to have been beaten.

(b)

It is easy for them to please John.


John is easy to please.

COMPARING ENGLISH AND CZECH SUBJECTS


Czech

(a)
(b)

pro-drop language (Subjects can be expressed by a verbal morphology).


The preferred Subject is the element carrying the Agent semantic role, i.e.
Agents have a strong tendency to become and remain Subjects.

English

(a)
(b)

not a pro-drop language (Subjects must be overt free morphemes)


The semantic role is often ignored and Subject is often purely syntactic.

(8)

The door closed.


Bonnie seems to read the book.

(a)
(b)

(a')
(b')

Dvee se zavely.
Zd se, e Jan te knihu.

39

7.2
(9)

Exercises
EXERCISE ===========================================

Discuss what is the Subject of the following sentences. Following the table0on page
40 give all possible semantic, morphological (Case, agreement) and syntactic (word order
and syntactic processes) reasons for your analysis. If necessary, use substitution.
(a)
(b)
(c)
(d)
(e)
(f)
(g)

My little brother will read the letter soon.


This is a cat on the mat.
The point is that we are leaving.
It is obvious that we need more equipment.
Whoever wants to go can go.
In the hole there is a string.
Who can be interested in chemistry?

(h)
(i)
(j)
(k)
(l)
(m)
(n)

Can Mary help you?


There is a cat on the mat.
That we need it is obvious.
It was Peter who arrived late.
All I did was hit him on the head
There comes the bus.
Help him!

(10) EXERCISE ===========================================


Using thje same criteria as in the Excercise abve, i.e. those in the table 0 on page 40
discuss whether the underlined expressions in the following sentences are Subjects?
Which properties of Subjects do they have and which do they miss?
(a)
(b)

Brutln italsk tok na Habe byl komentovn svtovm tiskem.


The brutal Italian attack to Ethiopia was commented by the world press.

(c)
(d)

Vidla svho bratra zamykat gar.


She saw her brother locking the garage.

(11) EXERCISE ===========================================


Referring to the Exercise (21) above find the elements which express the semantic
roles of the underlined Verbs. Are they expressed in a standard way? Explain the term
raising to Subject.
(a)
(b)

It is easy to catch a cold in autumn.


Some cold is always easy to catch.

(c)
(d)
(e)

People say that Mary has a new boyfriend.


It is said that Mary has a new boyfriend.
Mary is said to have a new boyfriend.

(f)
(g)
(h)

I believe that William has read the novel himself.


I believe him to have read the novel himself.
William is believed to have read the novel himself.

(i)
(j)

It is impossible for me to bear Theodor.


Theodor is impossible to bear.

(k) It appears that they sold out the book.


(l) It appears that the book has been sold out.
(m) The book appears to be sold out.

40

(12) EXERCISE ===========================================


(a)
(b)
(c)
(d)

Each constituent in a sentence can be defined w.r.t. several criteria:


Categorial label (NP, AP, VP, etc)
Semantic Role (Agent, Patient, Goal, Beneficiary, Instrument, etc)
Syntactic (sentence) Function (Subject, Object, Attribute, Adjunct, etc)
Pragmatic (discourse) Role (Theme/Topic, Rheme/Focus)
S
XP

The boy
(a)

NP

(b)

Agent

(c)

SUBJECT

(d)

Theme/TOPIC

Aux/Mod

VP

must

YP

read

a book.
NP

CZ (Agent = Subject)

Patient
OBJECT

EN (Subject = Topic)

Rheme / Focus

Using the above concepts describe the underlines expressions.


(a)
(b)
(c)
(d)

Mary was working on her paper.


It has been raining for the whole week.
Mary seems to know everything.
It is said that John has left.

(e)
(f)
(g)
(h)

The paper has been written in time.


It is impossible for John to get there.
People say that John has arrived.
Reading is said to be esay.

(13) EXERCISE ===========================================


Discuss the specific properties of the Subjects in the following sentences.
Some formal equivalents of the English structures in (13) above will be ungrammatical,
find them and explain the distinction referring to specific characteristics of Subjects.
English
(i) Open the window!
(ii) Thank you very much.
(iii) Came late and watched TV.
(iv) It became dark.
(v) There is a book on the table.
(vi) I saw him do it.
(vii) My brother/He was awarded a prize.
(viii) Henry/He is sure to come.
(ix) This book reads well.
(x) Your journal is a pleasure to write for.
(xi) It is a pleasure to write for you.

Imperative
Ellipsis
Diary style
Weather werbs
Existential/locative structures
Raising to Object (amalgamate)
Passivization
Raising to Subject
Mediopassive
Passivization
Expletive/Linking it

41

ATTRIBUTE

H&P (2002) pp. 526-561, QGLS (2004) pp. 402-437; Dukov (1994) pp. 484-517;
H&P (2005) pp.112-121;
(1)

NOUN PHRASE ==> Head Noun + its Modification


e.g. some very interesting German journals about geography

All what is subordinated to N (all parts of NP) is attribte.


The order of modifiers (scope)

(2)

the/my

favorite

hairy

specific Det/Poss/APs

shepard

dogs

general APs + secondary As

-N-

The scope of adjective covers all elements within NP au to the head N. The initial modifiers
are therefore the most specific.
(3)

(a)
(b)

the [big black dog]


the big [black dog]

8.1

Prenominal Attributes

(1)

DETERMINERS

(c)
(d)

a/the book
* that some book
takov njak kniha
each/every/any/some/no man
which/what man

(a)
(b)

(2)

POSSESSIVE NPs

(a)
(b)
(c)
(d)
(e)

(*a/the)our Peter's book


(a/the) bird's nest
* the dress's colour
[NP my older brother's] book
[NP the earth's] gravity

(3)

AP PREMODIFIERS

(a)
(b)
(c)
(d)

a [APvery large] [AP dark green] book


true gentlemen
a great Italian painter
a Spanish student

42

(4)

'DERIVED/SECONDARY ADJECTIVES'
(morphological Nouns (in productive compounds))

(f)

exercise book
steel wire
corrosion immunity
the UEFA Cup
three act play
* three acts play
heat-resistant glass lamp shades

(a)
(b)
(c)
(d)
(e)

(5)

ADVERBS

(a)
(b)

the then president


the down train

(6)

PARTICIPLES

(a)
(b)

a forgotten artist
an entertaining person

(7)

QUOTATIONAL COMPOUNDS
(a) a take-me-as-I-am smile
(b) an after-dinner cigar

8.2

Postnominal Attributes

(8)

POSTNOMINAL ADJECTIVES

(a)
(b)
(c)
(d)

Romance A's
idiosyncratic A's
A's follow pronouns
complex (postmodified) A's

(9)

PREPOSITIONAL PHRASES

Princess Royal, court martial, attorney general


syntax propper,
something [AP very interesting]
man [AP taller than me]
hero [AP faithfull to his ideals]

(a)
(b)
(c)
(d)
(e)
(f)
(f)

the book [PP of my brother]


a man [PP of courage]
a pair [PP of trousers]
a great variety of opinions
a way to school
a girl with a blue scarf
a story about animals

(10) INFINITIVES

(a)
(b)

a promise to help
a man to talk to

(11) -ING FORMS

(a)
(b)

the book lying on the table


the man reading newspaper

(12) CLAUSES

(a)
(b)

the book which you gave me


a warning that nobody should enter

43

(13) APPOSITION

(a)
(b)

The River Danube


Paul Smith the lawyer

(14) COMPLEMENT (Secondary Predication / Small Clause)


(a)
(b)
(c)
(d)
(e)

He was born blind.


(He was born + he is blind )
We came home tired.
He ranks high.
She dyes her hair black.
It's enough to drive me mad.

8.3

The Relation between the Noun and Its Attributes

(a)
(b)
(c)

Semantic/argument role (includes Possession)


Modification (specification, generalization, characterization, etc)
and also Determination, quantification (Determiners)

(15)

(a)
(b)

Their attack, the attack of wild tribes


Benjamin's house, the house of my grandma

(16)

(a)
(b)

a red rose, an intelligent student,


a house with a red roof

While verbal semantic roles are usually clear and arguments are realized w.r.t. their
interpreation, semantic roles related to Nouns are often ambiguous and their interpretation
depends of context.
(17)

(a)

Our Mary's picture...


The picture of Mary.

(b)

Maruin obrzek.
Obrzek na Maruky.

( Possessor / Agent / Patient)

The combination of more co-occurring modifiers (especially possessives and of-phrases),


can, however, restrict the interpretation.
(18)

8.4

(a)
(b)

Mary's picture of Renoire.


Maruin obrzek Renoira.

Exercises

(19) EXERCISE ===========================================


Underline the most salient interpretation(s) of the bold modifier. (Poss = possessor)
(a)
(b)
(c)
(d)
(e)

Tmhle vis ten Jaruin obrzek.


Tmhle vis obraz Picassa.
Tmhle vis obraz vaeho strce.
Poslouchali Mariinu / Shakespearovu bse.
Poslouchali marinskou / Shakespearovskou bse.

( Poss / Agent / Patient)??


( Poss / Agent / Patient)??
( Poss / Agent / Patient)??
( Poss / Agent / Patient)??
( Poss / Agent / Patient)??

44

(20) EXERCISE ===========================================


Compare the characteristics of the underlined Attributes. Consider their
(i) distribution (i.e. their position w.r.t. the head N and other pre-/post-modifiers)
(ii) uniqueness (How many of such elements can modify one head N?),
(iii) complexity (Are the Attributes Ns, NPs or PPs?),
(iv) semantics (Is the range of possible forms restricted to some meaning only?),
(v) interpretation (i.e. their ability to carry some specific semantic role w.r.t. the head
N),
.
(a) (*that) Johns party
(d) citys life
(b) that John party
(e) the city life
(c) that party of John(s)
(f) the life of the city
(i)
(ii)
(iii)
(iv)
(v)

........................................................................................................................................
........................................................................................................................................
........................................................................................................................................
........................................................................................................................................
........................................................................................................................................

(21) EXERCISE ===========================================


Assuming the following structure of NP what is the Czech equivalent of English
secondary adjective N/A? Use the following examples and give some more. (Comment the
bracketing paradox.)

(Q) - D/POSS - (Q) - A A - N/A (a)


(b)
(b)
(c)

- of.P - PP - Vinf/ing - clause

city towers
down town
new hospital building
English history teacher

(22) EXERCISE ===========================================


Based on the following examples discuss the distinctions English and Czech in the
field of pre-/post-/central Determiners. Give more examples concetrating on
(i) number and obligatoriness of determination,
(ii) order of determinrs and adjectival modifiers,
(iii) position of general quantifiers and numerals.
(a)
(b)
(c)
(d)
(e)
(f)
(g)

The book.
The blue book
The book of his.
The three boys.
All the three boys.
Those nice intelligent energetic boys.
famous Italian actor

(a')
(d')
(d')
(d')
(d')
(d')
(g')

*Blue book.
*Blue the book.
*The his book.
*Three the boys.
*The all three boys.
?Those nice energetic intelligent boys.
??Italian famous actor

45

(23) EXERCISE ===========================================


Assuming the structure of NP as in the scheme (21) above, discuss the distinction
between the Czech and English Attributes in the postnominal position. Give examples.
(i)

what is the Czech equivalent of English of-phrase

..................................................................................................................................................
..................................................................................................................................................
(ii) what is the Czech equivalent of English PP postmodifiers
..................................................................................................................................................
..................................................................................................................................................
(ii) what is the order of the postnominal modifiers
..................................................................................................................................................
..................................................................................................................................................
(24) EXERCISE ===========================================
Discuss the properties of the underlined elements. What is their category? Give
reasons for your claim. What is it 'bracketing paradox'?
(a)

the local government policy

(b)

the money exchange rate

(25) EXERCISE ===========================================


Give examples of English APs of the suggested complexity.
(a)
(b)
(c)
(d)

bare AP [AP A]
.
AP with premodified A, [AP -- A]
AP with postmodified A [AP A -- ]
AP with both pre- and post-modified A

....intelligent...
.......................................................................
.......................................................................
.......................................................................

(26) EXERCISE ===========================================


Which of the AP in (25) above can be used in the following functions? Consider all
a/b/c/d options. On the following line give examples of 'b' and 'c' and mark their
grammaticality.
Premodifying Attribute

a/b/c/d

(b)......................................................(c)....................................................................................
Postmodifying Attribute
a/b/c/d
(b)......................................................(c)....................................................................................
Nominal Predicate after Copula a / b / c / d
(b)......................................................(c)....................................................................................
Subject Complement
a/b/c/d
(b)......................................................(c)....................................................................................
Object Complement
a/b/c/d
(b)......................................................(c)....................................................................................

46

NEGATION

H&P (2002) pp.785-850, QGLS (2004) pp. 775-799; Dukov (1994) pp.336-348;
H&P (2005) pp.149-158; G &Q (1991) pp. 223-230;
9.1

Kinds of Negation

9.1.1 Semantic Negation


Opposites (=pairs of words which are distinct in at least one feature) are part of each
natural language. The phenomena possibly reflects some psychological property of a
human brain (the abbility to see the facts as contrastive or in binary opposition)
(1)

(a)
(b)
(c)
(d)

good
day
friend
mother

vs.
vs.
vs.
vs.

bad
night
enemy
father

9.1.2 Partial Negation (Lexical, phrasal)


The first stage of grammaticalization of negation. Some specialised grammatical morpheme
(affix) is used to negate a unit
(a) a word (e.g. un-, in-, dis, -less, -free)
(b) a phrase (not)
a)
(2)

Lexical negation
(a)

im+possible, un+easy, dis+proportionate, non+sense


coffein+free, speech+less

(b)

It is il-logical.

(not a negative clause, X = Y)

Scales of positive grammatical negation opposite


(3)
b)
(4)

(a)
(b)

true untrue -- false


easy uneasy difficult

Phrasal Negation
The particle not negates the whole phrase (usually some existing sentence member)
(a)
(b)
(c)

He prepared the dinner in the living room, not in the kitchen.


Mary not her little sister will drive the car today.
I want to read novels not to study vocabulary.

The negated phrase can be also the VP. To negate the whole clause, we must negate the
sentence modality (the Predicate).

47

9.2

Sentence negation

CLAUSAL NEGATION (propositional, 'grammatical') negates the clause, i.e. the whole
proposition. In English it is done by a particle not added to the Mod/Aux or by a negative
polarity item with a scope over the Predicate.
(5)

(a)
(b)
(b)
(c)
(d)
(e)

Mary will not help you.


Mary will never help you.
Nobody can do it for you.
I met nobody in the park.
?? I gave the book to nobody.
??? I met Jessica never / nowhere.

(particle not)
(negative quantifier in ADV)
(negative quantifier in SUBJ)
(negative quantifier in OBJ)
(partial negation?)
(partial negation?)

Partial (lexical and phrasal) negation can combine with the clausal negation.
(6)

(a)
(b)
(c)

It is impossible.
It is not possible.
It is not impossible.

(positive clause)
(negative clause)
(negative clause (positive meaning))

(7)

(a)
(b)
(c)

I was allowed not to come.


I was not allowed to come.
I was not allowed not to come.

(positive clause)
(negative clause)
(negative clause (positive meaning))

(8)

Scalar reading of lexical negation vs. opposites.


(a)
(b)
(c)
(d)

(9)

It is true.
It is easy.
He is tall.
She is beautiful.

It is not true.
It is not easy.
He is not tall.
She is not beautiful.

It is false.
It is difficult.
He is short.
She is ugly.

INTENSIFICATION:
... at all, no X whatever, not one ...

(a)
(b)
(c)

He is not big at all.


I have no money whatever.
Norbert has not a single friend.

9.3

The form and number of negative operators

A negative sentence can contain more negative operators (constituents denoting empty
sets). In fact any sentence member can be a negative operator. Compare the following
examples.
SUBJ, ADV, OBJ,

(10)
(a)
(b)
(c)
(d)
(e)

OBJ,

Petr vera Maruce tu knihu ne-dal.


Ni-kdo vera Maruce tu knihu ne-dal.
Ni-kdo ni-kdy Maruce tu knihu ne-dal.
Ni-kdo ni-kdy ni-komu tu knihu ne-dal.
Ni-kdo ni-kdy ni-komu ni-c ne-dal.
48

(11)

(a)
(b)
(c)
(d)
(d)

Peter did not give the book to Mary.


Peter did not give any/*no-thing to Mary.
Peter did not give any/*no-thing to any/*no-body.
No-body gave any/*no-thing to any/*no-body.
*No-body did not give ...

Czech: Negative Concord ('Multiple' negation)


English: Unique negation
(12)

NEGATIVE/POSITIVE QUANTIFIERS
no-, not any- / some- + -body/-thing/-where/-time/-how...

complex nature of pronouns = polarity item + 'proform'


Polarity elements are sensitive to the polarity of the context
(=their interpretation depends on the linguistic context).
Negative polarity items: appear only in the scope of negation.
9.4

The Choice of the Unique NEG in English

See also above in (11) on page 49.


(13)

(a)
(b)
(c)
(d)
(e)
(f)

Nobody has any friends.


* Anybody has no friends.
Adam and Mary do not have any friends
Adam and Mary have no friends.
Adam will never come.
Adam won't ever come.

As early as possible / as short as possible... Negative polarity items any MUST be in the
scope of negation. (also ever, at all, etc).
9.5

Position of Negation in English (revision)

9.5.1 Declarative sentences


Clausal negation in English is formed with the help of a grammatical element not. Its
position is fixed and represents an important point in the sentence structure so it is to be
stated precisely.
(14)

(a)
(b)
(c)
(d)
(e)
(f)

David can NOT be reading.


?David can be NOT reading.
*David can be reading NOT.
*David NOT reads.
??David reads NOT.
David does NOT read.

49

Negative particle (+negative/short Adverbs) follows the first Aux/Mod

(15)

David

SUBJECT

can

NOT

- - - read

will
might
is (-ing)
'does'

never
just

phonologically empty
nonemphatic do becomes lexicalised
to 'provide DO-support'
for the NOT particle

Mod/Aux

NEG

- - - VERB

The importance of the POSITION, the first phonetically present Mod/Aux distinct from
other Auxs and VLEX. This element precedes the particle not (bound morpheme -n't).
(The only exception is the Copula/lexical Verb be.)
(16)

Complex Predicate in English


will / have /

(a) read
(b) look
(c) take

at / for
care (of)

modality + Tense + Aspect


(a) lexical Verb
Mod / Aux
(b) phrasal Verb
particle
(c) light Verb
nominal part of the Predicate

functional level
lexical level.
NEGATION
(not/never/+NEG)
9.5.2 Negative Questions
ad (i)

Inversion: Recall which element (how many of them) inverts with a Subject?

(17)

(a)
(b)

ad (ii)

Insertion of not or -n't

David will be reading.


*Will be David reading?

(c)
(d)

Will David be reading?


*Will be reading David?

Consider the categorial status of English not and its morpho-phonetic contraction -n't
as signalled by the grammaticality judgements below. It shows that -n't is a bound
morpheme, while not is a particle: requires the presence of Mod/Aux but is not
morphologically unified with it.
(18) (a)
(b)
(c)
(d)

David won't be reading.


Won't David be reading?
David doesn't read.
Doesn't David read?

(a')
(b')
(c')
(d')

David will not be reading.


* Will not David be reading?
David does not read.
* Does not David read?

50

9.6

NOT vs. NEVER

The distinctions between not and never can be stated as a distinction between a
grammatical morpheme (not) and lexical morpheme (never: Adverb: mayor open class
category). In a sentence this distinction results is signalled by distinct behaviour.
A.
(19)

Sentence function (and interpretation)


(a)
(b)

(tomorrow = Adverbial of time)


He will not arrive tomorrow.
?? He will never arrive tomorrow. (2 Adverbials of time?)

Sentences like above appear only in colloquial language (defensive style).


B.
(20)

C.
(21)

D.
(22)

Position (=after (the first?) Mod/Aux)


(a)
(b)
(c)
(b)

He will not / won't be reading.


*He will be not reading. (= VP negation)
He will never be reading.
He will be never reading.

DO-support (and position)


(a)
(b)
(c)
(c')
(b)

He does not / doesn't read.


* He not / n't reads.
* He does never read.
? He never does read.
(pragmatically special)
He never reads.

Question formation (Inversion in front of the Subject)


(a)
(b)
(c)
(d)
(e)
(f)

Does HE not read?


*Does not HE read?
*Does HE n't read?
Doesn't HE read?
Does HE never read?
*Does never HE read?

Recall that b/f are similar, in both cases the structure is ungrammatical because more than
one "word" is inverted with Subject. (*Does often he read such books?)
E.

Fronting (Topicalization,WH questions)

(23)

(a)
(b)
(c)
(d)

To Mary Josephine never has written.


The letter Joe did not receive.
Yesterday William did not arrive.
When did Barbara arrive? (inversion)

(24)

(a)
(b)
(c)
(d)

Never will he read.


? Never he will read.
*Not will he read.
*Not he will read.

51

F.
(25)
G.
(26)

Usage in Separation
Will he read?

(a)
(b)

- Never!
- *Not. / *N't. / No.

Interferring inflection
(a)
(b)

*He do not read-s / stop-ed.


He never reads / stoped.

The categorial status of English never, not and -n't can be expressed in the terms of the
distinction between
grammatical elements

lexical element

bound morpheme - particle -

free morpheme

-n't

9.7

not

never

Exercises

(27) EXERCISE ===========================================


In both the following examples the negative element (-n't, never) appear in front of
the Subject. Discuss the distinctions.
(a)
(b)

Won't you help Mary?


Never will I help Mary.

(28) EXERCISE ===========================================


Recall the definitions of (a) Question formation and (b) position of negationin
Standard English. Then mark and explain the (un)grammaticality of the following
examples.
(a)
(b)
(c)
(d)

Won't Benjamin help you?


Will Benjamin help you not?
Will Benjamin not help you?
Will Benjamin help you never?

(e)
(f)
(g)
(h)

Will not Benjamin help you?


Will often Benjamin help you?
Will Benjamin never help you?
Will never Benjamin help you?

(29) EXERCISE ===========================================


(i)
(ii)

Which of the following sentences are positive/negative? Why?


Translate the sentences to English and try to keep the same kind of negative element.

(a)
(c)
(e)
(g)
(d)

Je inteligentn.
Nen inteligentn.
Mm hodn neptel.
Nemm hodn neptel
Mm v Praze jenom jednoho kamarda.

(b)
(d)
(f)
(c)
(e)

Je hloup.
Mm hodn ptel.
Nemm hodn ptel.
Nemm v Praze dnho kamarda.
Znm v Praze st vc ne 5 lid.

52

(30) EXERCISE ===========================================


Negate the underlined constituents and show Czech morphemes creating the lexical
and phrasal negation. The clause should remain positive! Then translate the sentences to
English and do the same.
(a)
(b)
(c)
(d)
(e)
(f)
(g)
(h)

Petr je mj ptel.
Ta cesta je bezpen.
Nov fakta jsou relevantn.
Maminka dnes spchala do koly...
Pijdu dnes.
Stran m bav pracovat.
Pikzal mu odpovdat celou vtou.
Sed v kuchyni.

.......................................................................
.......................................................................
.......................................................................
.......................................................................
.......................................................................
.......................................................................
.......................................................................
.......................................................................

(31) EXERCISE ===========================================


(i)
(ii)

Which part of the sentence is to be negated in Czech, to get a clausal negation? What
is the morpheme? Consider the following sentences.
Translate the sentences to English and do the same.

(a)
(b)
(c)
(d)

J chodm do koly.
J jsem chodila do koly.
J budu chodit do koly.
J bych chodila do koly.

.......................................................................
.......................................................................
.......................................................................
.......................................................................

(32) EXERCISE ===========================================


Can the partial negation combine with the clausal negation in Czech in the same way
as it can in English? Try it for the sentences like in (6)/(7) on page 48 and discuss the
interpratation.
(33) EXERCISE ===========================================
Decide about the polarity of the sentences below and then discuss the interpretation
of the polarity element.
(a)
(b)
(c)
(d)
(e)
(f)

Somebody will help you at last.


Do you want some more tea?
Did anybody arrive in time?
Anybody can help you, in fact.
I do not know anybody.
I know nobody.

(34) EXERCISE ===========================================


Compare the interpretation of the polarity element any-/some- and state the
interpretative rule for ANY-.
(a)
(b)
(c)

I can give you anything.


I cannot give you anything.
Can you give me anything?

(a') I can give you something.


(b') I cannot give you something.
(c') Can you give me something?

53

(35) EXERCISE ===========================================


(i) fill the bold columns with the polarity specification [+], [-], [] or so...
(ii) in the central column relate the Czech equivalents of the English polarity items.
(iii) write the rule for interpretation of 'any'
English
SOME
ANY
NO

Czech
Nbody / thing / where

kdo / c(o) / kde


NI-

ANY - .....................................................................................................................................
................................................................................................................................................
(36) EXERCISE ===========================================
Which kind of negation can you find in the following examples? Discuss the STYLE!
(a)
(b)
(c)
(d)
(e)
(f)

The product is not faultless.


He doesn't like doing nothing.
Nobody has never got into trouble.
I don't know nobody.
Nobody ain't at home.
I never did it yesterday.

(37) EXERCISE ===========================================


Translate to Czech and discuss the distinctions concerning negation.
(a)
(b)
(c)
(d)
(e)
(f)

There is no question of Harrys decoration not being earned.


You cannot say it was nothing; it cant have been nothing.
She hardly saw anything.
Anyway, you cant do just nothing.
He went there not because of Mary.
He didnt go there because of Mary.

(38) EXERCISE ===========================================


Translate the following sentences. Which of them are examples of partial negation
and which are negative sentences? Can you prove it?
(a)
(b)
(c)
(d)
(e)

It is impossible, isn't it.


Anybody can not speak.
He feels better with nobody, doesn't he.
Mary likes no work, doesn't she.
Mary likes no work, does she.

54

10

THE SCOPE OF NEGATION

H&P (2002) pp.785-850, QGLS (2004) pp. 775-799; Dukov (1994) pp.336-348;
H&P (2005) pp.149-158; G &Q (1991) pp. 223-230;
The domain in which the polarity item any gets negative interpretation.
In English it is usually a simple clause. The following examples show that the scope is not
enlarged to other clause in a complex/compound sentences.
(1)

See the interpretation (negative vs. arbitrary) of any

(a) Main-Subord.

Bonnie cannot see anything [ when anything covers his eyes.


= free choice / * negative

(b) Subord.-Main

Though w did not ask anybody [ any people can come.


= free choice / * negative

(c) Main-Main

I did not bring anything but [ you can take any of those.
= free choice / * negative

The scope of negation is signaled by the interpretation of the negative polarity any- .
(In the above examples any is a free choice and not negative, which signals that the
negative operator has a scope over one simple clause only.)
10.1 Shortening the scope of NEG
Though negation takes scope over the whole simple clause, sometimes it is necessary to
restrict it (e.g. to get general, non-negative interpretation of the any- element)
(2)

(a)
(b)
(c)

Nebavm ses nikm.


Nebavm se s nkm.
Nebavm se s kmkoliv.

(3)

(a)
(b)
(c)

I do not talk to anybody.


I do not talk to somebody.
I do not talk to just anybody.

10.2 Enlarging the scope of NEG (NEG-Transportation/Transfer)


In complex sentences containing Verbs of 'think' type in the main clause, the negation can
be transported/transferred to the initial main clause.
(4)

he is NOT here.

(a)

I think

(b)

I do NOT think he is here.


main clause

subordinate clause

NEG Transportation: The shift/move of NEG from the embedded clause to the main
clause, the interpretation does not change in (a) and (b), but formally, the scope of the
negation covers both main and subordinate clauses in (b).

55

(5)

(a)

I think [[ that Mary will NOT help any-body/*some-body /*no-body at all.

(b)

I do NOT think [[ that Mary will help any-body/*some- body /*no- body at all.

In (a) the underlined main clause is positive. The negative particle not is in the
second/subordinate clause and the second/subordinate is negative, as signalled by the
negative interpretation of any-body and the presence of 'at all'
In (b) the main clause contains the negative particle not (although it is probably not
interpreted as negative, it still is formally marked as negative). The second/subordinate
clause does not contain any negative particle, but the negative interpretation of any-body
and 'at all' signal its negative polarity. The scope is thus enlarged into both cluases.
(6)

(a)
(b)

I think that Henry will NOTt write any-thing/*some-thing/*no-thing at all.


I do NOT think that Henry will write any-thing/*some-thing/*no-thing at all.

10.3 Negative Adverbs / Partial Negation


Appart from negative Adverb never, which creates "full" clausal negation, there are
Adverbs in English, which constitute so called "partial" clausal negation: e.g. hardly,
scarcely, barely, rarely, seldom, little, few The fromal characteristics of a polarite item
cannot be derived referring to semantics of the proposition but with the help of fromal
signals, namely with the form of
(a) related polarity item(s)
(b) question tag
(c) inversion when neg. adverb is fronted. Consider the following examples.
Compare the positive vs. negative patterns represented by 'often' and 'never':
(7)

(8)

(9)

(a)
(b)

He often knows nothing.


He never knows anything / *nothing.

... positive pattern


... negative pattern

(c)

He knows hardly/barely anything / *nothing.

... negative pattern

(a)
(b)

He often knows all, doesn't he /*does he.


He never knows, does he / *doesn't he..

... positive pattern


... negative pattern

(c)

He knows hardly/barely all, does he/ *does he?

... negative pattern

(a)
(b)

Often I will help you. / *Often will I help you.


*Never I will help you. / Never will I help you

... positive pattern


... negative pattern.

(c)

*Hardly I will help you. / Hardly will I help you.

... negative pattern

10.4 More Polarity Items


Apart from polarity elements containing items some-/any-/no-, there are more lexical
(phrasal, idiomatic) expressions sensitive to the scope of negation (and/or interrogative
operator).

56

(10) (a)
(b)
(c)

Maruka nehnula ani prstem.


*Maruka hnula ani prstem.
*Hnula Maruka ani prstem?

(11) (a)
(b)
(c)

Mary did not see a single friend / nobody on her trip.


*Mary saw a single friend/ nobody on her trip.
*Did Mary see a single friend/ nobody on her trip?

(a)
(b)
(a)

Maruka nehnula nim.


*Maruka hnula nim.
*Hnula Maruka nim?

(12) Some English polarity items


Both NEG and Q
any, anyyet, ever, long
need, dare
at all (UK)
give a damn
move an inch

only NEG
a single N (US)
either
?at all (US)
lift a finger,
spend a red cent
spend a penny,

Only Q

Never NEG

whether, how

already, too
sort of, kind of

what the hell

10.5 Exercises
(13) EXERCISE ===========================================
(i) Explain the process of NEG-transportation and show what signals the scope of
NEG in the folowing English examples
(a)
(b)

I believe that Mary will not say anything at all to anybody.


I do not believe that Mary will say anything at all to anybody.

(ii) Write Czech equivalents of the above sentences and check the scope of
negation. Underline the elements which show the polarity of the Czech sentences. Is the
process of NEG transportation the same in Czech as in English?
(a) ..........................................................................................................................................
(b) ..........................................................................................................................................
(14) EXERCISE ===========================================
(i) Explain what signals the presence of NEG scope in the following examples. Why
are the Adverbs like hardly/scarcely etc. called (partially) negative?
(a)
(b)
(c)

He does not help to anybody, does he.


He never helps to anybody, does he.
He hardly helps to anybody, does he.

(ii) Write Czech equivalents of the above sentences and check the scope of
negation. Underline the elements which show the polarity of the Czech sentences. Assuming

57

that 'partial' negation can refer to interpretation and formal characteristics, to which
extend can the Adverbs like hardly/st be called 'negative' in English/Czech?
(a) ..........................................................................................................................................
(b) ..........................................................................................................................................
(c) ..........................................................................................................................................
(15) EXERCISE ===========================================
Explain the (un)grammaticality of the following examples.
(a)
(b)
(c)
(d)
(e)
(f)
(g)
(h)
(i)
(j)

He knows nothing at all.


% Did you drink at all at the party?
*Did you have a single drink at the party?(US)
*He knows everything at all.
He doesnt know anything either.
*Did you drink either at the party?
*He knows everything either.
He doesnt understand it yet/*still.
Did you *yet/still go to the party?
He *yet/still remembers everything.

(16) EXERCISE ===========================================


Explain the (un)grammaticality of the following examples.
(a)
(b)
(c)

I sort of dont have any family.


I dont sort of have any family.
I (*dont ) have sort of a family

(17) EXERCISE ===========================================


Explain the phraseological vs. literate interpretation of the phrase lift a finger.
(a)
(b)
(c)

He hasnt ever lifted (??left) a finger for anybody, as far as I know.


?? Would you please lift a finger for me?
?? I often lift a finger for my friends.

(18) EXERCISE ===========================================


Consider the standard position of negative adverb never compared with with the position of
Adverbial/Adjunct of Time in the afternoon.
(a)
(b)

Joe will never do it.


Joe will do it in the afternoon.

(ii) Assuming that Adjuncts can be fronted in English, what do the following examples
say about the place from which the Adjuncts are moved (which of them trigger Aux-Subj.
inversion?)

58

(c)
(d)

In the afternoon Joe will do it.


Never will Joe do it.

(iii) Clausal negation in English, can result also from structures with negated Object or
Adverbial, though esp. the latter structures remain often ambiguous between (i) the clausal
and (ii) partial negation.
(e)
(f)

I have no friends.
= I do not have any friend.
I will do it in no time. = (i) I will not ever do it.
(ii) I will do it [ in short time].

... clausal negation


... partial negation

Recall that if the negative adverb negates the whole clause, and if such an adverb is
fronted, it triggers inversion. The contrast between (g) and (h) should help you to explain
the distinct interpretation of (i) and (j).
(g)
(i)

Often Mary will come.


In no time Mary will come.
(=she comes soon)

(h)
(j)

Never will Mary come.


In no time will Mary come.
(=she never comes)

(19) EXERCISE ===========================================


Translate (or paraphrase) the following sentences. Notice that (a/b) are ambiguous,
while (c-f) are not. Referring to the discussion in the exercise (18) above, explain the
distinction in terms of partial vs. clausal negation.
(a) They are ready for no job.
(b) The food tastes well with no salt.
(c)
(d)
(e)
(f)

For no job are they ready.


With no job they are ready.
With no salt the food tastes well.
With no salt does the food taste well.

.......................................................................
.......................................................................
.......................................................................
.......................................................................
.......................................................................
.......................................................................
.......................................................................
.......................................................................

(20) EXERCISE ===========================================


Find signals of negation and discuss their interpretation. Explain the
ungrammaticality.
(a)
(b)
(c)
(d)

Petr is friendly / unfriendly.


Petr is not friendly / unfriendly.
It is impossible for Peter to be unfriendly.
It is hardly possible for the chips to be fatless.

(d)
(e)
(f)
(g)
(h)

Peter knows nothing.


Peter does not know anyting / ?? nothing.
Peter does not know nothing.
No people can help you.
*No peple cannot help you.

59

11

ENGLISH SENTENCE PATTERNS (STEP BY-STEP)

H&P (2002) pp. 851-946 ; QGLS (2004) pp. 49-5,


H&P (2005) pp.159-173; G &Q (1991) pp. 159-1739
Word order in English simple clause is grammaticalized ("fixed"). Students often use the
'SVO(MPT)' pattern. In the following section 11.1 we are revising the topic step by step,
arguing that though 'SVO' is a right typological characteristics of English constituent order
(especially when compared with other languages), it is not the way to capture the
constituent order in various English sentence patterns.
11.1 Analytic characteristics of the English Predicate
The "Verb" in English often consists of several parts. Distribute the words/phrases/units
(=constituents) into the table below. Consider also the size of the elements.
(1)
(2)
(3)
(4)

Henry will sleep.


The boy is reading a novel.
The most beautiful girl is walking around our school every day at 5oclock.
He can put the book on its place as soon as Mary tells him where it should be.
S
(subjekt)

M/A
(Modal/aux.)

"VP"
'Verb'

V-complementation

(1)
(2)
(3)
(4)
11.1.1

Negation

Forming a clausal negation in English, where is the position of negation (particle "n't/not")?
Consult the examples and define the position precisely with respect to the elements
(columns) in the above table. Mark the relevant line dividing the relevant columns.
(5)
(6)
(7)

Henry will not sleep.


The boy is not reading a novel.
The most beautiful girl is not walking around our school every day.

11.1.2

Polar Questions:

Polar question in English are formed by inversion/fronting. Study the examples and define
the fronted element precisely with respect to the elements (columns) in the above table.
Circle the relevant column and mark by arrow the process of inversion/fronting.
(8) Will Henry sleep?
(9) Is the boy reading a novel?
(10) Is the most beautiful girl walking around our school every day around 5 oclock?

60

Assuming that the position of negation and the question inversion are the two main criteria
for dividing the English Predicate field, distribute also the constituents in the following
structures into the table below.
(11) Henry will be sleeping well.
(12) The new boy will look for you tomorrow.
(13) She may have been having shower for more than two hours.
fronting in
questions

position of NOT

S
(subjekt)

M/A
(Modal/aux.)

'Verb'

"V VP"
V-complementation

(11)
(12)
(13)
Analytic Predicate in English
The above data illustrate what is called analytic Predicate = a Predicate which can be
divided into several parts. Such a Predicate is characteristic of Modern English (contrary to
Czech which has a synthetic Predicate). Notice (check/show) that the English word order
expressed as S-V-O, would not be able to capture with clarity neither the position of
negation nor the question inversion (and it presuposes the presence of Object, which is not
always present). Therefore we will replace it by S M/A V (O), where M/A +V
represent the two main parts of the English analytic Predicate.
11.1.3

Verb 'be' and single lexical Verbs

Apparent exceptions: There are cases which seem distinct from S M/A V (O)
The Verb be
Consider the following examples with "be". Locate the Verb "be" in the table below using
the position of negation and the question inversion. Try to make some meaningful
generalizations.
(14)
(15)
(16)
(17)
(18)
(19)
(20)

Henry will be reading the book.


We have been students for many years.
He may have been at home.
Henry is reading the book.
We are students.
I am to go.
Dont be silly!
S

M/A

"V-VP"

(14)
(15)
(16)
(17)

61

(18)
(19)
(20)
Conclusion: the Verb "be" (Auxiliary/Copula/existential but not Modal) is special, because
it can be a V, but in the absence of Modal/Auxiliary it takes its position leaving V empty.
(Is there any distinction between "be" when (i) Aux, (ii) Copula, (iii) existential, (iv) Mod?)
'Lexical' Verbs
In English, the 'lexical' Verb is often preceded by an Auxiliary or modal element, but
sometimes such a Verb appears alone. Assuming our division between A/M and V, fill into
the slots in the table the constituents in the following examples. Take into account the
criteria we have been using i.e. the position of negation and the question inversion.
(How do we call the "do" in (24) , (25), (26)?)
(21)
(22)
(23)
(24)
(25)
(26)

Henry does sleep at home.


He did read the novel you gave him.
Elisabeth does walk around our school every day around 5oclock.
Henry sleeps at home.
He reads the novel you gave him.
Mary walks around our school every day around 5 oclock.
S

M/A

"V-VP"

(21)
(22)
(23)
(24)
(25)
(26)
DO-Support (Covert Auxiliary DO)
To keep the idea of English constituent order as general as possible, we can claim that
English Predicate is always analytic and assume that in non-emphatic positive declarative
clause the Auxiliary DO remains covered (phonetically unrealized). It becomes overt when
needed (e.g. emphatic context, questions, and negative particle not > require do-support)
11.1.4

Other use of the structure S-M/A-V-(O)

The word/constituent order in English has a grammatical function. We must state the
order oc constituents in a clause right to be able to discuss main types of sentences, state
properly the position of negation, the process of inversion and also other processes.
'SVO' does not mean, that "the first word is a Subject."
Remember:

i)
ii)
iii)
iv)
v)

there can be topicalised constituents in front of the Subject


SUBJECT can be very complex (it is NP, not N!)
MOD/AUX is one word,
Verb can be complex (contain other Auxiliaries) and
V-complementation varies according to the Verb.

62

(27) Constituent order in English


SUBJECT - MOD/AUX - (NEG) -

VERB

the position of the polarity (positive/negative)


(= after the first Mod/Aux)
Question inversion (= the first Mod/Aux inverts in front of the Subject)
Referring to the scheme in (27), we can e.g. define which elements appear in English
question tags, short Yes/No answers and in questions of surprise.
(a)

Question tags

(28) Henry will sleep, wont he?


(29) The boy is not at home, is he?
(30) The most beautiful girl walks around our school, doesnt she?
Question tags are formed with
(b)

Short answers

(31) Will Henry sleep?


(32) Does the most beautiful girl walk there? (33) Is the boy at home?
-

Yes, he will.
Yes, she does.
No, he is not.

/*Yes, Henry will.


/*Yes, she walks.

Short Yes/No answers are formed with


(c)

Questions of surprise

(34) Henry will sleep.


(35) The girl walked around our school ?
(36) The boy is not at home?

Will he?
Did she?
Isnt he?

/*Will Henry?
/*Walked she?

Questions of surprise are formed with


11.1.5

Negative questions

Stating the rule for question inversion in English in (27), negative questions show the
importance of the size of the inverting element. Consider the following examples and
explain the ungrammaticality. (Underline the inverting element. How many words invert?)
(37) (a)
(b)
(c)
(d)

Henry wont arrive.


Henry will not arrive.
The girl is never reading novels.
The girl is often reading novels

Wont Henry arrive?


Will Henry not arrive?
Is the girl never reading novels?
Is the girl often reading novels?

(38) (a)
(b)
(c)

Henry will often arrive


Henry will never arrive
Henry will not arrive.

* Will often Henry arrive?


* Will never Henry arrive?
* Will not Henry arrive?

63

11.2

WH questions

What is the constituent order in the WH question in English? Compare the (a) examples in
(39)/(40) with the (b) examples and try to generalize the changes.
(39) (a)
(b)

Lucy will read the book tomorrow.


Intelligent students can learn a lot of exciting facts at school every day.

(40) (a)
(b)

What will Lucy read?


What can intelligent students learn at school every day?
S

(39)
(39)
(40)
(40)

M/A

"V-VP"

a
b
a
b

(41) The main characteristics of the WH question is

(a) a question inversion,


(b) a fronted WH element

The characteristics of the WH element


Consider which part of the declarative sentence is questioned in the WH questions.
a) Underline the interrogative element (+ which sentence member it is)
b) Consider what would be the answer (+ which sentence member it is)
Try to answer the following questions:
- How complex is the initial element?
.......
- Which structures do not show the inversion? ...
(42)
(43)
(44)
(45)
(46)
(47)
(48)

What can intelligent students learn at school every day?


Which facts can intelligent students learn at school?
How many exciting facts can intelligent students learn?
Where can intelligent students learn a lot of exciting facts?
When can intelligent students learn a lot of exciting facts?
Who can learn a lot of exciting facts at school every day?
Which students learn a lot of exciting facts at school every day?

There are some other structures which suggest that the fronting of some constituent is a
more general phenomena. Consider the similarity of the following examples.
(49) When/Where/how will Henry read the novel?
(50) Never will Henry read the novel.
(51) Never did she help me.
Fronted operators (negative/interrogative constituents with special reference)
can trigger the inversion.

OPERATOR -

Mod/Aux

SUBJECT

- - - VERB
SUBJ-Mod/Aux Inversion

e.g. WH- fronting

64

11.2.1

Indirect WH questions and Echo Questions

There are some special WH questions which show characteristics more/less distinct from
those we stated above. Compare the following structures with the "standard" in (39)/(40),
i.e. concentrating on (a) the presence of inversion, (b) the position of the WH element.
Indirect WH questions
(52) He asks what intelligent students can learn at school every day.
(53) He asks which facts intelligent students can learn at school every day.
(54) He asks whether intelligent students can learn a lot of exciting facts at school.
Indirect WH questions are special because they .
Echo questions
(55)
(56)
(57)
(58)

Intelligent students can learn what at school every day?


Intelligent students learn a lot of which facts at school every day?
Intelligent students learn a lot of exciting facts every day where?
NOTICE > Who can learn lot of exciting facts at school every day?

Echo WH questions are special because they .....


11.3 Imperatives (Covert Subject)
The structure S M/A V (O) suggests, that Subject is a necessary part of the English
clause. Sometimes, however, it is not overt. Consider the following examples of
imperatives and look for the signals which show the presence of Subject. (Make similar
examples to prove the claim.)
(59) (a)
(b)
(c)
(d)
(e)

Open the window!


(You, sob) dont do it!
Help yourself!
Help him, will you!
Lets go, shall we.

What is the distinction between (a) and (b) in the following example? Explain the
ungramaticality of (d).
(60) (a)
(b)
(c)
(d)

Let us go, shall we.


Let us go, will you.
Lets go, shall we.
*Lets go, will you.

(61) Indirect orders


(a)
(b)

I asked Mary to open the window.


I ordered him to go.

65

12

SUBORDINATE CLAUSES

H&P (2002) pp. 851-1096 ; QGLS (2004) pp.985-1146; Dukov (1994) pp. 588-647;
H&P (2005) pp.159-194; G &Quirk (1991) pp.262-335;
12.1 Classification of Subordinate Clauses
Consider the sentence functions of the bold elements.
(1)

A
(a) EVELYNE
(b) EVELNA

gave
podala

B
C
A PROPOSAL to the commitee
NVRH
pedsednictvu

A.
B.
C.
D.

Subject
Object
(Prep.) Object
Adverbial

(2)

(A) WHO(EVER) WANTED gave a proposal to the commitee .


KDO CHTL podal nvrh pedsednictvu.

D
yesterday.
vera.

Subordinate Subject Clause


Subordinate Object Clause
Subordinate Object Clause
Subordinate Adverbial Clause

(B)

Evelyne gave them WHAT THEY WANTED.


Evelna jim podala CO CHTLI.

(C)

Evelyne gave a proposal to who(ever) she wanted.


Evelna podala nvrh
komu chtla.

(D) Evelyne gave a proposal to the committee after she introduced herself..
Evelna podala mvrh pedsednictvu
jakmile se pedstavila..

Subordinate clauses have the function of some sentence member of its main clause.
Two kinds of Classification of subordinate clauses
1.

The traditional way: According to the sentence member the clauses replace/realize.
See (1) and (2) above.

2.

Dukov etc.: According to the part of speech which typically occurs in the same
function.

A.
B.
C.
D.

Subject
Object
(Prep.) Object
Adverbial

i)
ii)
iii)

NOMINAL content clause ('replace main NP sentential memebers')


ADVERBIAL content clause ('replace Adverbials')
ATTRIBUTIVE content and RELATIVE clauses ('replace Attributes')

Noun
Noun
Noun
Adverb

Nominal Content Clause


Nominal Content Clause
Nominal Content Clause
Adverbial Content Clause

66

12.2 Nominal Content Clauses


Used in the function of a nominal phrase (Subject, Object, Complements, etc).
Connected with the main clause with a connecting element: 0, THAT, WH-ELEMENT
(3)

(a)
(b)
(c)
(d)

12.2.1
(4)

He's told us jokes.


He's told us THAT he couldn't come.
This story is interesting.
WHAT you say is interesting.

= Object / Noun
= Object / Nominal Content Clause
= Subject / Noun
= Subject / Nominal Content Clause

Resumptive element ('oprn vraz')


(a)
(b)
(c)
(d)
(e)
(f)
(g)
(h)

IT is interesting WHAT you say.


(*It/*That) What you say, is interesting.
TO je zajmav, CO k.
(To,) Co k, je zajmav.
He was interested in (*it/*that) what he needed.
Zajmalo ho jen to, co poteboval.
(Ten) Kdo et, (ten) m za ti.
(To) O em nev, (to) t nebol.

English vs. Czech equivalents: clause vs. semiclause


(i)
(j)

Vad mi, e to udlal. Chci, aby to udlal.


I mind his doing it.
I want him to do it.

Pro-form replacing the whole proposition: so


(5)

Can we get there in time? (a) I hope (that) we can get there in time.
(b) I hope (*that) so

12.2.2

Tense/Mood (and other characteristics) of the Subordinate Clause...

The distinction between the main and subordinate clause can be seen also in the level of
their separate and independent referrence to Time/mood. Some subordinate clause loose
their independence and their form is influenced by some characteristics of the the main
clause (and/or by a Preposition).
(6)

(a)

Indirect speech

He said he would do it when he came.


ekl, e to udl a pijede.

(b)

Indirect question:

He asked if/whether he would come.

(c)

Subjunctive:

It is appropriate
essential
necessary
not fitting

that he SHOULD come.

67

12.3 Adverbial Content Clauses


Greenbaum & Quirk (1990) pp.262-335; Dukov (1994) pp. 588-647;
Quirk/Greenbaum/Leech/Svartvik (2004) pp.985-1146;
12.3.1

Kinds of Adverbial content clauses

Used in the function of Adverbial (Adjunct)...


(7)

Place:

(a)
(b)

I found my notebook WHERE I LEFT IT.


WHEREVER WE CAME, we meet with friendliness.

(8)

Time :

(a)
(b)

I'll give it to you, AS SOON AS I (*WILL) FINISH.


WHEN (I AM) ON HOLIDAY I (will) dismiss all work.

(9)

Manner (and condition): There exist many kinds of Adverbials of 'Manner', e.g.
cause, reason, purpose, result, condition, concession... Include also
conditional clauses.

(a)
(b)
(c)
(d)
(e)
(f)
(g)
(h)

His Voice broke AS IF HE WERE GOING TO CRY.


There is no light here SO THAT WE COULD WORK.
We had to go there so BECAUSE WE WERE ASKED TO.
I am not going to go there IN CASE YOU DON'T COME.
He will do it, IF YOU ASK HIM.
IF YOU ASKED HIM he would do it.
(AL)THOUGH THE WEATHER IS FANTASTIC we will have to stay at home.
SINCE SHE DID NOT ANSWER we can do it our way.

Replace Adverbial of Place. Provide information about location or


direction to the contents of the main clause.

Replace Adverbial of Time. Provide temporal information w.r.t. the


contents of the main clause. (In English they do not have future Tense.)

(10) Manner (Grading): Adverbial clauses of Manner can be also related to Adjective or
Adverb (Grading, degree, comparison, etc)
(11)
(a) Mary is more lazy THAN ANYBODY CAN IMAGINE.
(b) He was so stupid THAT I COULD NOT BELIEVE IT.
(c) The airplane got as high AS NOBODY HAD EXPECTED.
12.3.2

Conditional Clauses

(a)

Real

(i) classification w.r.t. Time

(b)

Unreal

(ii) classification w.r.t. interpretation

(a)
(b)
(c)

Future
Present
Past

68

12.3.2.1 Future Conditional Clauses


(12) = always real (it can happen)
(a)
(b)

If it rains (tomorrow) I WILL not WORK.


Jestli bude pret (ztra), NEBUDU PRACOVAT.

12.3.2.2 Present Conditional Clauses


(13) Real

(a)
(b)

If it rains (now), they ARE wet.


Jestli pr (te), tak JSOU mok.

(14) Unreal

(a)
(b)

If it rained (now), I WOULDN'T BE here.


Kdyby prelo (te), tak BYCH tu nebyl.

12.3.2.3 Past Conditional Clauses


(15) Real

(a)
(b)

If it rained (yesterday), they DIDN'T MANAGE to do it.


Jestli prelo (vera), tak to NESTIHLI udlat.

(16) Unreal
(a)
(b)
(c)

If it had rained (yesterday), he WOULDN'T HAVE GONE there.


Kdyby (bvalo) prelo (vera), (NEBYL) BY tam EL.
Had it rained (yesterday), he WOULD HAVE TAKEN an umbrella.

12.3.2.4 Mixed Patterns


e.g. PAST condition with PRESENT result
(17) Real

(a)
(b)

If it rained (yesterday), the barrel is full of water (now).


Jestli (vera) prelo, tak je ndr pln vody (te).

(18) Unreal

(a)
(b)

If they had done it (yesterday) they wouldn't be busy (today).


Kdyby to bvali udlali (vera), nemli by spoustu prce (dnes).

12.4

Relative Clauses

The referential quality of the nominal element can be RESTRICTED (modified) by


many ways. The 'categorial' status of the Attribute, i.e. the element specifying the meaning
of the Noun can range from a minimal Determiner to a complex clause. (Revise the
properties of the individual Attributes, see section 8 on page 42.
(19) (a)
(b)
(c)
(d)
(e)
(f)
(g)
(h)

a/the book
my older brother's book
a large green book
exercise book
the book of my brother
the book lying on the table
the way to go
the book which you gave me
69

RELATIVE CLAUSES are used in the function of Attributes, i.e. they modify a nominal
element which is present in the main clause. They are connected with the main clause with
a connecting element: 0, THAT, WH-ELEMENT
12.4.1

Kinds of Relative Clause

(A) RESTRICTIVE (who/which, that, )


(20) (a)
(b)
(B)

NON-RESTRICTIVE (who/which, *that, *)

(21) (a)
(b)
(c)
(d)
(C)

Mm (krom jinch) dva ptele, kte mi p pravideln.


I have (besides others) two friends (*,) who write me regularly.

Posledn host, kter odeel, byl Mr. Brown.


The last guest, who left, was Mr. Brown.
Mm dva ptele, kte (= a ti) mi p pravideln.
I have two friends */, who (= and they) write me regularly.

FREE (QUASI-RELATIVE CLAUSES) (who/which/wh-, *that, *)

(22) (a)
(b)
(c)
(d)
12.4.2

Piel Petr, kter (= a ten potom) pozdravil.


Peter came , who (= and then he) said hello.
Dal mu drek, co (= a to) m irituje.
He gave him a present, which (= and this) irritates me.
Relative Clause Formation

Consider the 'fusion' of two propositions in 0 and (24) below..


(a)

Compare the content of the (i) two independent propositions and (ii) the complex
sentences with a relative clause in (a) and (b) of 0 and (24).
Mary loves ONE BOY.

(23)

(a)

Mary loves

(b)

THE BOY
the head
of the RC

THE BOY
the head
of the RC

WHO-M
relative
pronoun

THE BOY is over there.


WHO
relative
pronoun

Mary loves

is over there .

- original
position

is over there.

70

William showed me A GIRL

(24)

THE GIRL
the head
of the RC

(a)

William showed me

(b)

I saw the mother of THE GIRL


the head
of the RC

I saw THE GIRL'S mother.


WHO-SE mother
relative
pronoun

I saw

- - .

WHO-M William showed me - - .


relative
original
pronoun
position

Consider the importance of the sentence function of the NP replaced by a relative pronoun
(for the deletion and morphology of the pronoun).
12.4.3

Relative Pronoun Deletion

Relative pronoun in English can be deleted if the relative clause is structurally complete
(25) (a)
(b)

Give me the letter (that) Hilary sent you.


Give me the book *(that) was sent to you by Hilary.

Omitting that in (25)(b) would violate the rule about obligatory Subjects in English.
(26) (a)
(b)

I met the man (who) we were talking about.


I met the man about *(who) we were talking.

Omitting who in (26)(b) would violate the rule about Prepositions [_NP].
(27) (a)
(b)

I met the girl *(whose) mother we were talking about.


I must buy that book the name *(of which) I heard on TV.

Attributes whose/of which in (27) are optional and hence non-recoverable. Must be overt.
12.4.3.1 Preposition stranding in Relative clauses
Recall that English has complex (analytic) verbal Predicates and as one of the
consequencies it allows preposition stranding (see section 5.2.1 on page 25).
In the sentence (28) below both the preposition about can be analysed as belonging to
either the verb 'talk about' or as a part of the PP about the girl.
(28)

Peter talked about the girl.


(a)
(b)

Peter talked [PP about the girl ]


Peter talked about [NP the girl ]

71

The analysis (b) in (28) results in preposition stranding in (i) WH questions


(ii) Passive structures
(iii) Relative clauses
About whom did Peter talk?
Who did Peter talk about?

(29)

(a)
(b)

(30)

The girl was frequently talked about by Peter.

(31)

This is the girl who is frequently talked about.

Preposition stranding influences the Case-marking. Case in English remains overt


especially if the Case-marked element appears immediately after its Case assigner. If the
distance is further, the morphology often desappears.
(32)

(a)
(b)

12.4.4

the man about


the man

%WHO/WHOM/*THAT/*0 you learned


WHO/%WHOM/*THAT/0 you learned about

Content Nominal/Attributive vs. Relative Clauses

Discuss the distinction between the element 'heading' the subordinate clause.
+ NOMINAL Content Clause
what you mean.
co m na mysli.

(33)

I can see
Chpu

(*that/*it)
(to)

(34)

I can see
Vidm

n
(the fact)
(to)

+ ATTRIBUTIVE Content Clause


(that) he is ill.
e je nemocn.

(35)

I can see
Vidm

N
the book
(tu) knihu,

+ RELATIVE Clause
that/which John gave me.
kterou mi dal John.

Notice that the distinction is not in the subordinate clause itself. The clauses are classified
in a different way because the element which they are related to is distinct:
(a)
(b)
(c)

in (33) it is the Verb (optional presence of expletive, linking it)


in (34) it is an NP which however can be deleted and its meaning is either expressed
in the Verb
already or it is contained in the subordinate clause
in (35) it is NP (usually definite) which is a full-fledged constituent (the relative
clause adds meaning but is not syntactically needed.

72

12.5 Exercises (final revision)


(36) EXERCISE ===========================================
Change the underlines constituents into subordinate clauses. How would you classify
the clauses?
(a)
(b)
(c)
(d)
(e)
(f)

Mary arrived yesterday.


The man reading newspaper is my brother Tom.
I like Jospehine.
Joe is not interested in sports.
Don't say that!
He is more tall than Mary.

(37) EXERCISE ===========================================


Are the following sentences grammatical?
(a)
(b)
(c)
(d)

He will help you when he will come.


He will ask Connie when she will come.
Mary will arrive if she will have time.
Bill wants to know if she will have time.

(38) EXERCISE ===========================================


In which contexts English does not tolerate 'will/shall' (Czech future)? What is it
'Tense Shift'? State in detail the conditions when it applies (and exceptions to the rule).
(39) EXERCISE ===========================================
(i)
(ii)

State which kind of subordinate clause is the underlined text.


Underline the sentence member in the main clause which is related to the subordinate
clause (what is the syntagma?).
(iii) Replace the subordinate clause by a non-clausal constituent.
(a)
(b)
(c)
(d)
(e)
(f)

I wonder where he put it.


This shows you I was right.
The material world is the only world there is.
Bill sat at the same table as Fred (did).
He hit him so hard that he tumbled off the chair.
I'll give it to whoever you suggest.

(40) EXERCISE ===========================================


Write the example of a complex sentence containing
(a)

a nominal content clause in the function of Subject (with and without the linking it)

...................................................................................................................................................
(b) a nominal content clause in the function of Object
...................................................................................................................................................

73

(c)

a nominal content clause in the function of Object of Preposition

...................................................................................................................................................
(d) a nominal content clause in the function of Predicate
...................................................................................................................................................
(e) an Adverbial content clauses in the function of Temporal adv. and adv. of Place
...................................................................................................................................................
(f) an Adverbial content clause in the function of Grading
...................................................................................................................................................
(g) an irreal conditional clause in past
...................................................................................................................................................
(h) an Adverbial content clause in the function of Adverbial of Reason
...................................................................................................................................................
(41) EXERCISE ===========================================
(i) Look into the table below and notice that some subcategorized Complements can/must
be expressed by a clause. See the variety of the Complements, add more examples.
(ii)

What are the semantic roles of the PP in (9) and (18)?

(iii) What is the distinction between the second NP in (10) and (16)?
VERB
monotransitive

ditransitive V

complex
transitive V

subcategorization
V, [--NP]
V, [-- that-CL]
V, [-- wh-CL]
V, [-- wh-INF]
V, (-- to-INF]
V, [-- ing-CL]
V, [-- NP to-INF]
V, [-- NP ing-CL]
V, [-- NPd PP]
V, [-- NPi NPd]
V, [-- NPi that-CL]
V, [-- NPi wh-CL]
V, [-- NPi wh-INF]
V, (-- NPi to-INF]
V, [-- NPd AP]
V, [-- NPd NP]
V, [-- NPd adv]
V, [-- NPd PP]
V, [-- NPd to-INF]
V, [-- NPd bare-INF]
V, [-- NPd ing-CL]
V, [-- NPd -ed-CL]

example
1. Jane read a diary.
2. I think that we have met.
3. Can you guess what she said?
4. I learned how to sail the boat.
5. We've decided to move to Prague.
6. She enjoys playing squash.
7. They want Jane to help us.
8. I hate Jane shouting.
9. He wrote a letter to Jane.
10. He told Jane a story.
11. They told Jane that I was ill.
12. He asked Jane what time it was.
13. Mary showed Jane what to do.
14. I advised Jane to see the doctor.
15. That music drives Jane mad.
16. They called Jane idiot.
17. I left Jane at home.
18. I put the book on the shelf.
19. They knew Jane to be a spy.
20. I saw Jane leave the room.
21. I heard Jane shouting.
22. They got Jane killed.

74

(42) EXERCISE ===========================================


(a)

a declarative positive sentence with V, [_ NP]


.......
(b) an interrogative (polar) positive sentence with V, [_ NP, NP]
.......
(c) a declarative negative sentence with V, [_ PP]
.......
(d) a WH question with V, [_ NP PP]
.......
(e) a declarative negative sentence with Aux/V, [_ NP]
.......
(g) a declarative negative sentence with V, [_ ]
.......
(43) EXERCISE ===========================================
Distribute the constituents from the following sentences to the scheme below. Use the
position of negation (NEG) as a criterion for distribution.
(a)
(b)
(c)
(d)
(e)

Henry will call his mother.


His mother never answers.
I've got several very interesting papers.
Barbara has a ery nice house.
Mary is lazy.
S
NP
[Subject]

Mod/Aux (NEG)

VP
[Predicate]
V

XP

(44) EXERCISE ===========================================


Explain the Agreement of the English relative pronouns. Compare with Czech.
(a)
(b)
(c)
(d)
(e)

This is the book WHICH/THAT/ I like to read.


This is the book WHOSE author I met in Prague.
He has two brothers neither of WHOM is present here.
I received several books all of WHICH are useless.
He is still the keen sportsman *WHO/THAT/ he's always been.

75

(45) EXERCISE ===========================================


Underline the subordinate clauses and classify them . Use both functional and
categoria terminology.
(a)
(b)
(c)
(d)
(e)
(f)
(g)

I did not know that you would arrive sosoon.


That Clara will do it was clear from the very beginning.
The said that if I helped him he would be able to finish it by noon.
Because the world is round one caanot see America from Spain.
It is as I predicted.
He figured that they all must have left already.
Barbara has moremoney than you can imagine.

(46) EXERCISE ===========================================


Many students classify the subordinate clause according to the element which
appears at the beginning (conjunction, pronoun). But consider the following examples.
(a)
(b)

It was the time he spoke about his love to Cecile.


She said he spoke about his love to Cecile.

(c)
(d)
(g)

When Mr. Brown arrived I helped him.


Mary asks when he arrived.
I need to know the exact time when he arrived.

(e)
(f)

William must have passed the exam if he really tried.


Nobody knows if he really tried.

(47) EXERCISE ===========================================


Classify the following sentences and make a schematic picture of the relations.
(a)

As soon as I see him I will tell him that the work must be done by Friday if he wanted
to leave for a weekend.

(b)

If you want I will show you the building much taller than those you have in Chicago.

(c)

I wanted to do something about it and I thought I would have a chance because


before we decided to go home on Sunday evening, Sheila said that Bill was out but
when we arrived he was sitting in the living room and he rejected to leave because he
wanted to watch TV and so we finally ended up in the pub and nothing happened.

(d)

It is unlikely that John would help you because lately he is very tired when he arrives
home and goes to bed after he finishes his dinner and so I think you will have to finish
the work yourself as quickly as you can because it is sure that the teacher repeated
several times that in this case the deadline is fixed and there will be no excuses and
whoever delivers the assignment late than it is the date written on the web page will
have to repeat the course because he will not give him the credit.

76

13

COMPLEX SENTENCE ANALYSIS

H&P (2002) pp. 43-70, QGLS (2004) pp. 35-92; H&P (2005) pp. 11-28;
THE FOUR LEVEL SENTENCE ANALYSIS
Each element belongs to a specific
A. PART OF SPEECH (paradigmatic categories).
In a sentence it receives
B. SEMANTIC ROLE
C. SENTENCE FUNCTION (syntagmatic relations),
In the discourse it receives
D. PRAGMATIC FUNCTION
13.1 Parts of speech (word categories)
Part of speech
1. Nouns (common/propper, concrete/abstract,
countable/mass, etc),
2. Adjectives (primary/secondary, central/peripheral),
3. Pronouns (demonstrative, personal, possessive, reflexive,
reciprocal, interrogative, relative, indefinite, negative),
4. Numerals (cardinals, ordinals, etc),
5. Verbs (Auxiliaries, Modals, lexical),
(causatives, ...of sensual perception, ...of movement)
6. Adverbs,
7. Prepositions,
8. Conjunctions (sub/co-ordinate),
9-11. Interjections, Particles, Articles

usual function
Subject / Object
attributes
Subject / Object / Attribute
attribute / Adverbial
Predicate
Adverbial
part of Adverbial, Object
-----

13.2 Semantic Roles: Participans of the Verbal Event/Arguments of the Verb


semantic role
AGENT
Experiencer
Force
Theme
Patient
AFFECTED OBJECT
Cause
Result of the action
Locative
Agentive
Possession
Instrument
RECIPIENT /
Beneficiary

example
(1) John killed a rabbit
(2) John saw a rabbit
(3) The frost killed the rabbit
(4) John sleeps. The boat sung.
(5) John killed the rabbit
(6) He overturned the chair.
(7) I love John.
(8) He wrote a book.
(9) John climbed Mount Everest.
(10) The room dances two couples easily.
(11) We (have) got many letters.
(12) They threw stones.
(13) I read the book to/for Mary
(14) I gave Mary the book

realization
SUBJECT

direct OBJECT

indirect OBJECT

77

SPACE
TIME
Process
Respect
Contingency (cause,
reason, purpose, result,
condition, concession)
Modality
Degree
Modification
Determination
Intensification

(15)
(16)
(17)
(18)
(19)
(20)
(21)
(22)

The rabbit ran to the forrest.


The rabbit ran very far.
I am still in Pilsen.
The job was done with an axe.
The job was done by John.
I helped him with the rabbit.
I did it because I wanted to.
If he comes I will tell you.

ADJUNCTS Adverbials

(23) I certainly/probably did it.


(24) I badly need it.
(25) Little John is here.
ATTRIBUTE
(26) The/this girl is Mary.
(27) He is very/extremely/pretty big.
CONJUNCT
(28) He runs very/extremely quickly.
Grading / Comparison (29) This boy is more/the most
handsome.
Selection and obligatory character of SUBJ- OBJ- ADJt, free relation of ADJt-DISJt...
(1)

(a)
(b)
(c)

Elisabeth reads THE BOOK TO MARY.


Jonathan runs THE RACE / A MILE / A SHOP.
Bush relies ON NUCLEAR WEAPONS / VERY MUCH / THIS YEAR

Standard and less standard realization


(2)

(a)
(b)

JOHN (=Agent) / THE KEY (=Instrument) opened the door.


THE DOOR (=Patient) was opened BY JOHN (=Agent).

13.3 Sentence Functions (sentence memebers, syntagma)


syntagma/couple with function
Subject
Predicate
Noun
Predicate
attribute
determination
quantification
Verb
Subject
Object (direct /
indirect)
Adverbial / Adjunct
Verb = Copula*
Verb + Noun
(Subject)
Verb + Noun (Object)
adjective/Adverb
*Copulas: (i)

nominal Predicate
Subject Complement
Object Complement
Grading subjunct

example
(a) John was sleeping.
(b) little John
(c) the book
(d) three boys
(e) to read the book to John
to read John a book
(f) to go to school quickly, every day, at
five o' clock
(g) He is an idiot / handsome / at school
(h) John came back tired
(i) to paint the door green
(j) very tired, very quickly, as big as he,
much bigger than me

current (be, seem, appear, feel, look, sound, taste), (ii) resulting (become, get, grow, turn, prove)

78

The characteristics of prototypical / unmarked SUBJECT


(1)
(2)

semantic roles
(a) morphology
(b) syntax

Agent (some variety including none)


Subject Case / NOM (if visible)
Agreement on Predicate (3sg,Pr)
NP (+ its proforms. Also PP, VP, clause)
Precedes the Predicate, inverts in questions,
appers in Question tags, short answers...
It is an antecedent to anaphor

The characteristics of prototypical / unmarked OBJECT


(1)
(2)

semantic roles
(a) morphology
(b) syntax

Patient/Affected Object (wide variety)


Object Case / ACC (if visible)
NP (+ its proforms. Also PP, VP, clause)
Immediately follows V (but also initial)
Structural Object can be passivised

13.4 Sentence Dynamism / Functional Sentence Perspective


Tomorrow my sister Mary will help John with his homework.
THEMATIC

Non-thematic

THEMATIC

TRANSITIONAL

RHEMATIC

TRANSITION transition
proper

RHEME

THEME
Proper

DiaTheme

RHEME
propper

(3)
XP

Aux/Mod

VP
V

This lady

can

help

XP
your son.

(a)

NP

(b)

Agent

CZ (Agent = Subject)

Patient

(c)

Subject

EN (Subject = Topic)

Object

(d)

Theme/TOPIC
non-dynamic

NP

Rheme/FOCUS
dynamic

79

Example

Our Mary will read the new book on Tuesday.


Our

Parts of speech

Sentence
functions

Semantic Roles

Mary

NOUN PHRASE
NP: Noun phrase
Pronoun
Noun
(possessive) (propper)
SUBJECT
Subject
Attribute
bare
Subject

will

read

Verbal complex
Verb
Verb
(Modal) (lexical)

Predicate

Modification
(possessive)

Agent

Action

Our

Mary

will read

the

new

book

on

Tuesday

Simple clause
VERB PHRASE
NP: Noun phrase
PP: Prepositional phrase
Article
Adjective
Noun
preposition
Noun
(definite)
(common)
(propper)
PREDICATE
Object (direct)
Attribute
Attribute
bare
Object
Determination Modification

the

new

Adverbial Adjunct
of Time

Patient

Temporal frame

book

On Tuesday

80

TOPICS TO DISCUSS
At the end of the course the students are expected to be able to discuss some basic grammatical concepts: to
show understanding of the main terminology and to provide relevant English examples illustrating their
classication and theoretical claims. The following topics are more or less similar to those which appear at the
final state exam (in the part concerning English language - grammar).

1.
Nouns. Explain how and why words are classified into parts of speech/word classes.
Describe in detail the category of Noun: give its classifications related to (i) semantics
(proper vs. common, abstract vs. concrete), (ii) morphology (derivational morphemes,
inflectional paradigms related to Countability/Number, Animacy/Gender, Case), and (iii)
syntax (the structure of NPs and their functions). Demonstrate each phenomenon with
prototypical examples and standard exceptions.
2.
Pronouns. Provide a classification of English pronouns, giving examples of each
group. Concentrate on properties of personal pronouns with respect to (i)
meaning/semantics, (ii) form/morphology and (iii) usage/distribution/syntax. Mention the
notion of anaphor (discourse and syntactic), and demonstrate this contrast showing the
distinction between English personal and reflexive/reciprocal pronouns.
3.
Verbs. Explain how and why words are classified into parts of speech/word classes.
Describe in detail the category of verbs in English: give its classifications related to (i)
semantics (dynamic vs. stative verbs), (ii) morphology (esp. inflectional paradigm related to
Tense, Aspect, Voice), and (iii) syntax (Auxiliary/Modal/Lexical verbs, and classification
based on verbal subcategorisation). Demonstrate each phenomenon with both prototypical
examples and standard exceptions.
4.
Clause structure. Start with the logical structure of a clause, demonstrating the
concept of verbal event, and its participants (classification of the main semantic roles).
Demonstrate for both active and passive patterns how the logical structure is formally
realised in English into a clause (i.e. show the variety of frequent correlations between
semantic roles and sentence members).
5.
Subjects. Explain briefly but generally the concept and classification of syntagmatic
relations/sentence members/functions. Demonstrate in more detail the function of Subject
in English. Describe its (i) semantics (semantic role(s)), (ii) morphology (Subject Case and
Agreement) and (iii) syntax (position in declarative, interrogative and imperative clausal
patterns). Explain with examples the function of English expletive subjects (there and it).
6.
Objects. Explain briefly and generally the concept and classification of syntagmatic
relations/ sentence members/ functions. Demonstrate in more detail the function of Object
in English. Describe its (i) semantics (semantic role(s)), (ii) morphology (Object Case) and
(iii) syntax (i.e. position with respect to the verb and to adjuncts). Describe the process of
passivisation, mentioning in detail the distinction between active and passive structures.
7.
Clausal word order. Demonstrate distinct sentence patterns in English. First, explain
the concept of operator (the first Modal/ Auxiliary), provide the orders for declaratives and
interrogatives and for affirmative and negated patterns. Then describe in more detail the
post-verbal part of a clause related to verbal subcategorisation (transitive vs. intransitive,
double objects, the order of object(s) and adverbials, etc.).

81

RELATED LITERATURE
The list A below gives practical manuals of English grammar which can help students not
fully familiar with the pratical usage of the structures discussed. The working knowledge of
this manuals is assumed for the course.
The list B provides bibliography for the more theoretical manuals covering the topics in
more detail. They provide some discussion of the phenomena, provide much more data and
demonstrate alternative terminologies and analyses.
The list C provides bibliography for the cited works and some additional literature related
to the topics discussed in the course.
A.

PRACTICAL MANUALS

Alexander, L.G. (1993): Longman Advanced Grammar. Reference and Practice. Longman.
Hewings, Martin (2005): Advanced Grammar in Use (2nd edition) with answers and CD
ROM. CUP.
Jones, Leo (1991): Cambridge Advanced English. Cambridge: Cambridge UP.
Leech, Geoffrey & Svartvik, Jan (1975) A Communicative Grammar of English.
Longman, London.
Murphy, Raymond (2004): English Grammar in Use With Answers and CD ROM : A
Self-Study Reference and Practice Book for Intermediate Students of English. 3rd
edition. CUP.
Svoboda, Ale (2004) Lectures on English Syntax. Filozofick fakulta Ostravsk
univerzity, Ostrava.
B.

THEORETICAL MANUALS

Dukov, Libue (1994) Mluvnice souasn anglitiny na pozad etiny. Academia Praha,
Prague.
Huddleston, Rodney and Pullum, Geoffrey K. (2002) The Cambridge Grammar of the
English Language. Cambridge University Press, Cambridge.
Huddleston, Rodney and Pullum, Geoffrey K. (2005): A Students Introduction to English
Grammar. Cambridge University Press, Cambridge.
Leech, Geoffrey (1971) Meaning and the English Verb. 3rd edition. Longman, London
2004.
Quirk, R., Greenbaum, S., Leech, G. & Svartvik, J. (2004) A Comprehensive Grammar
of the English Language. Longman, London
Quirk, R., and Greenbaum, S. (1991): A Students Grammar of the English language.
Longman1991.
C.

FURTHER RELATED / CITED LITERATURE

Akmajian, A., Demers, R.A., Farmer, A.K. & Harnish, R.M. (1990) Linguistics: An
Introduction to Language and Communication. MIT Press, Cambridge, Mass.

82

Crystal, David (1987) The Cambridge Encyclopedia of Language. Cambridge University


Press, Cambridge.
Demers, Richard A. & Farmer, Ann K. (1991) A Linguistics Workbook. The MIT Press,
Cambridge, Mass.
Firbas, Jan (1992) Functional Sentence Perspective in Written and Spoken
Communication. Cambridge University Press.
Fromkin, Victoria & Rodman, Robert (1990) Introduction to Language. HBJ.
Mathesius, Vilm (1915) 'O passivu v modern anglitin.' In Sbornk filologick 5. (pp
198-220)
Sgall, Hajiov, Burov (1980) Aktuln lenn vty v etin. Academia, Praha
Vachek, Jan (1959) Two Chapters on Written English. Brno Studies in English.
Veselovsk, Ludmila (2009) The Course in English Morphology and Morphosyntax.
Palacky University Press, Olomouc.

83

You might also like